Tuesday, August 13, 2019

The Problem with Quantum Measurements


Have you heard that particle physicists want a larger collider because there is supposedly something funny about the Higgs boson? They call it the “Hierarchy Problem,” that there are 15 orders of magnitude between the Planck mass, which determines the strength of gravity, and the mass of the Higgs boson.

What is problematic about this, you ask? Nothing. Why do particle physicists think it’s problematic? Because they have been told as students it’s problematic. So now they want $20 billion to solve a problem that doesn’t exist.

Let us then look at an actual problem, that is that we don’t know how a measurement happens in quantum mechanics. The discussion of this problem today happens largely among philosophers; physicists pay pretty much no attention to it. Why not, you ask? Because they have been told as students that the problem doesn’t exist.

But there is a light at the end of the tunnel and the light is… you. Yes, you. Because I know that you are just the right person to both understand and solve the measurement problem. So let’s get you started.

Quantum mechanics is today mostly taught in what is known as the Copenhagen Interpretation and it works as follows. Particles are described by a mathematical object called the “wave-function,” usually denoted Ψ (“Psi”). The wave-function is sometimes sharply peaked and looks much like a particle, sometimes it’s spread out and looks more like a wave. Ψ is basically the embodiment of particle-wave duality.

The wave-function moves according to the Schrödinger equation. This equation is compatible with Einstein’s Special Relativity and it can be run both forward and backward in time. If I give you complete information about a system at any one time – ie, if I tell you the “state” of the system – you can use the Schrödinger equation to calculate the state at all earlier and all later times. This makes the Schrödinger equation what we call a “deterministic” equation.

But the Schrödinger equation alone does not predict what we observe. If you use only the Schrödinger equation to calculate what happens when a particle interacts with a detector, you find that the two undergo a process called “decoherence.” Decoherence wipes out quantum-typical behavior, like dead-and-alive cats and such. What you have left then is a probability distribution for a measurement outcome (what is known as a “mixed state”). You have, say, a 50% chance that the particle hits the left side of the screen. And this, importantly, is not a prediction for a collection of particles or repeated measurements. We are talking about one measurement on one particle.

The moment you measure the particle, however, you know with 100% probability what you have got; in our example you now know which side of the screen the particle is. This sudden jump of the probability is often referred to as the “collapse” of the wave-function and the Schrödinger equation does not predict it. The Copenhagen Interpretation, therefore, requires an additional assumption called the “Measurement Postulate.” The Measurement Postulate tells you that the probability of whatever you have measured must be updated to 100%.

Now, the collapse together with the Schrödinger equation describes what we observe. But the detector is of course also made of particles and therefore itself obeys the Schrödinger equation. So if quantum mechanics is fundamental, we should be able to calculate what happens during measurement using the Schrödinger equation alone. We should not need a second postulate.

The measurement problem, then, is that the collapse of the wave-function is incompatible with the Schrödinger equation. It isn’t merely that we do not know how to derive it from the Schrödinger equation, it’s that it actually contradicts the Schrödinger equation. The easiest way to see this is to note that the Schrödinger equation is linear while the measurement process is non-linear. This strongly suggests that the measurement is an effective description of some underlying non-linear process, something we haven’t yet figured out.

There is another problem. As an instantaneous process, wave-function collapse doesn’t fit together with the speed of light limit in Special Relativity. This is the “spooky action” that irked Einstein so much about quantum mechanics.

This incompatibility with Special Relativity, however, has (by assumption) no observable consequences, so you can try and convince yourself it’s philosophically permissible (and good luck with that). But the problem comes back to haunt you when you ask what happens with the mass (and energy) of a particle when its wave-function collapses. You’ll notice then that the instantaneous jump screws up General Relativity. (And for this quantum gravitational effects shouldn’t play a role, so mumbling “string theory” doesn’t help.) This issue is still unobservable in practice, all right, but now it’s observable in principle.

One way to deal with the measurement problem is to argue that the wave-function does not describe a real object, but only encodes knowledge, and that probabilities should not be interpreted as frequencies of occurrence, but instead as statements of our confidence. This is what’s known as a “Psi-epistemic” interpretation of quantum mechanics, as opposed to the “Psi-ontic” ones in which the wave-function is a real thing.

The trouble with Psi-epistemic interpretations is that the moment you refer to something like “knowledge” you have to tell me what you mean by “knowledge”, who or what has this “knowledge,” and how they obtain “knowledge.” Personally, I would also really like to know what this knowledge is supposedly about, but if you insist I’ll keep my mouth shut. Even so, for all we presently know, “knowledge” is not fundamental, but emergent. Referring to knowledge in the postulates of your theory, therefore, is incompatible with reductionism. This means if you like Psi-epistemic interpretations, you will have to tell me just why and when reductionism breaks down or, alternatively, tell me how to derive Psi from a more fundamental law.

None of the existing interpretations and modifications of quantum mechanics really solve the problem, which I can go through in detail some other time. For now let me just say that either way you turn the pieces, they won’t fit together.

So, forget about particle colliders; grab a pen and get started.

---

Note: If the comment count exceeds 200, you have to click on “Load More” at the bottom of the page to see recent comments. This is also why the link in the recent comment widget does not work. Please do not complain to me about this shitfuckery. Blogger is hosted by Google. Please direct complaints to their forum.

460 comments:

  1. Einstein died before the experimental tests of EPR, namely Bell's theorem, showed that the very thing he wanted to complain about quantum theory, is actually correct. His insistence on a universe that is consistent with General Theory of Relativity turned out to bite him.

    Scientists are totally ok when pointed out that something fundamental is not understood. But you need to tell us exactly where/what it is, even if you cannot resolve it yet. It, however, cannot just be a preference.

    ReplyDelete
  2. Prof Sabine,

    What you are saying would point to a post-quantum theory. That is incredibly interesting. Might be Bohmian. Could you mention the relevant search terms, pretty please?

    ReplyDelete
  3. Andrew,

    In that case, you too are inventing opinions I don't hold and have never put forward. Congrats.

    ReplyDelete
  4. B.F

    I wish I could but as I said there's basically no one working on this, so there are no search terms. There's only me and my crappy blog and all the people who complain about it.

    ReplyDelete
  5. 'The obsession to "reduce" everything to "basic principles" ignores the potential presence of irreducible/strong emergent properties leading to unnecessary obfuscation and dead-ends.'

    In reality, emergent phenomena can be derived from fundamental basic principles. e.g. Helium-4 Bose-Einstein condensate is a highly different emergent phenomena of electrons and protons and neutrons.

    We are totally happy if you can give us some evidence for the breakdown of reductionism. But otherwise, we have no wish to go back to the days where serious learned people believed that biological systems are governed by this mysterious life force and hence can never be studied by scientific methods. That was the real dogma and orthodoxy. We have transcended that.

    ReplyDelete
  6. One of the Dirac-von Neumann postulates of QM assert that the state of the quantum system is a ray on the Hilbert space. This is the wavefunction that we are talking about.

    How is it that the state of a quantum system itself change as you update your mental model? Why should the quantum system care about your mental model? What is the state of the quantum system just before the measurement, before your mental model gets updated?

    ReplyDelete
  7. The many worlds people are asserting that the measurement contradictions are an artefact of choosing a stupid postulate. If you simply drop the collapse postulate and allow the most simple form of QM, including only Schroedinger evolution, the problems of measurement become smaller.

    Your assertion that it has to be an extension, adding new axioms, etc, is wrong.

    ReplyDelete
  8. Not sure what you are trying to do here.

    In modern parlance I suppose we can all agree that the basis of wavefunctions is a gauge choice. You could choose to always just work in terms of density operators, in which case the gauge invariance is trivial.

    Or in Einstein's parlance, absolute basis does not exist, and all quantum results should be relative to the emitter and observer bases only.

    So, what does this have to do with the conversation?

    I only nitpick that you should only assert that Aspect experiments et al reject _classical_ local hidden variables. You could have a _quantum_ local hidden variable, aka Bohm...

    ReplyDelete
  9. JeanTate,

    The experiments have been _done._ The 1960s called, and they want your talking point back.

    ReplyDelete
  10. Yes, if you concede that
    "The CI says there two distinct worlds, one quantum the other classical."
    then of course I would agree with
    "All interpretations impose auxiliary axioms or physical postulates."

    I disagree with:
    "However, we know from a measurement perspective QM has stochastic properties, even if the formalism of QM is deterministic."
    because I could use your own later words
    "More ψ-ontic interpretations such as MWI have a global wave function, but local observers are spontaneously carried along with one state or path in a decoherence set of states. There really is not much more explanation for why that appearance is so."
    ---as in, there is no need for more explanation, because I think that already successfully explained the problem to death. Of course, I do not claim that MWI explains the probability issue that they agree is an issue with MWI, I do think the decoherence arguments already satisfactorily sort out the illusion of collapse.

    As for
    'Bohmian QM has its issues as well, for with measurements the quantum potential has to be nonlocally “reset,” and Bohmian analysts often use standard QM to do this!'
    I am of the impression that Bohmian mechanics also can use decoherence, and hence the reset is not actually needed either. Of course, this might well be far from the standard practice. Nor does it solve the SR issues.

    So yes, I do agree with your that the overall picture is that every interpretation today has hidden mines. MWI seems to have trouble with probabilities (though below in another thread I was just asking Prof Sabine how my concrete calculation example of how a wavefunction under successive measurements would give the correct relative probabilities per branch, does not satisfy her requirements). Bohmian has to deal with SR.

    But I still maintain that at least some versions of Copenhagen is just plain wrong, as are GRW (again, because the experiments should be ruling them out as an entire class already). Of course, that does not mean that there is no defensible version of Copenhagen, and for intellectual honesty we always would need to acknowledge their existence.

    ReplyDelete
  11. JeanTate,

    The experiments already exist and are done.

    Even if they are not, trying to poke holes in the standard theory is, by definition, scientific. Michelson-Morley, by the textbook fake science history, is supposed to be about aether or not. In reality, it was about two different types of aether.

    If you wish to be closed-minded and ignorant of the issues, you are very welcome to exit the conversation. We actually care about knowing more about this, be it merely theoretical, or much more preferably, experimental.

    ReplyDelete
  12. JeanTate,

    Do not reverse the roles. You are in charge of proposing an experiment which is not compatible with QM and the Copenhagen interpretation (call it QBism or Psi-ontic if you like fancy terms).
    QM is a well-established theory. Plenty of experiments were made by Freedman, Aspect, Gisin, Gröblacher , Salart, etc. All these experiments ruled out the “hidden variables” scenario from 1972 to now.

    Another alternative: If you think that there is some sort of De Broglie-Bohm classical field then you must build some kind of potentiometer to measure it.

    As long as something like that is not presented, I’ll stick to the interpretation of the wave function as a complex distribution function.



    ReplyDelete
  13. And Sabine's point is that QM is not capable of *describing the observation of the measurement process.* Thus, her description of the measurement problem perfectly fits her acceptability criterion as a scientific problem and she is not being inconsistent at all.

    And if she is right that it is actually possible to *derive* that a solution to the problem would necessarily entail additional predictions beyond QM that could in principle be measured... well, that would be amazing and certainly scientific. The key question is what assumptions the derivation would require.

    ReplyDelete
  14. The wave length is NOT given by the (inverse of the) mass but according to the law of de Broglie by the (inverse of the) momentum.

    If we assume that the electron moves at a speed of 0.1*c , which is not too exotic, then the co-moving observer should see normal physics; so also physics according to the Schrödinger equation. But that is not the case.

    Because as said, in the rest frame of the electron and so in the rest frame of the co-moving observer the momentum is =0 and so the wavelength is infinite.

    The basic assumption of relativity (here SR) is that the same physical laws apply in any frame. And this is clearly violated here for the observer's frame.

    ReplyDelete
  15. B.F

    If one Believe in a Classical theory like the pilot wave we need a non-local action to explain the anti-corrleation of the singlet state. Right ?
    More generaly, you need non-locality to fake QM with a classical theory.

    ReplyDelete
  16. B.F. wrote: "The experiments have been _done._ The 1960s called, and they want your talking point back." This in answer to my question: "Can any of you propose an experiment (you know, the bedrock of science) whose results may move this discussion [on the measurement problem] forward? Even an in-principle experiment, one that is - today - impossible to do."

    Um, what experiments? Are you saying that the measurement problem cannot be addressed, in any way at all, by any new experiments? I'd like to be very clear about this.

    ReplyDelete
    Replies
    1. Sorry for the late reply. I didn't know the comment system suddenly works differently after a certain size limit.

      I think I should refer you to the comment on an earlier thread, where I have just replied to. Not the one with this quote you have taken.

      As to explain the particular comment quote, the story was that people used to dismiss all interpretation issues as experimentally untestable. The experiment that ended that, the only one in the 1960s, was Bell's.

      Part of the problem with proposing new experiments, however, is that it is difficult to come up with them in the first place. By your argument Pauli should not have suggested neutrinos, nor should Higgs suggest his boson.

      Not to mention that asking people to predict, say, what experiment would be the post-quantum epoch, is precisely the way to squander reputation points.

      Mind you, Couder silicone oil drop experiments show that it is plausible for a post-quantum theory to be pilot wave based. We won't know what it looks like at the moment, not least because the version I know of it still has SR issues. But qualitative features are already solved.

      If it is cleaner to consolidate our conversation here, please use this thread.

      Delete
  17. Hi Sabine,

    Ok, but I don't think it is true that Relational QM is psi-epistemic and as such I think your criticism(s) of psi-epistemic interpretations do no harm to Relational QM. Let's take them in order:

    1) 'The moment you refer to something like “knowledge” you have to tell me what you mean by “knowledge”, who or what has this “knowledge,” and how they obtain “knowledge.”'

    But Relational QM does not refer to "knowledge" of independent physical systems at all. It does not divide between the wave function as a real existing thing of an independent physical system (psi-ontic) or the wave function as mere "knowledge" or "information" about a real independent physical system. If it did, your criticism would be well deserved. No, in Rovelli's papers he is clear that only thing that exists is relative correlations. As he says, "Quantum mechanics is a theory about the physical description of physical systems relative to other systems, and this is a complete description of the world". The only *things* that *exist* and have physical meaning are the relations. The relations are not "knowledge" or "information" about independent physical systems, but rather the relations *are* the physical items themself.


    2) 'Referring to knowledge in the postulates of your theory, therefore, is incompatible with reductionism.'

    Relational QM is incompatible with the notion of absolute physical properties described in isolation from other physical systems. The universal wave function is a meaningless notion in Relational QM. But I don't see how this harms reductionism. And as above, Relational QM does not rely upon any definition of "knowledge."

    3) 'This means if you like Psi-epistemic interpretations, you will have to tell me just why and when reductionism breaks down or, alternatively, tell me how to derive Psi from a more fundamental law.'

    Again, I don't think Relational QM is actually psi-epistemic or at least not in the ways that are harmed by your criticism.

    This is why I'm really interested in your derivation of the problem as you mentioned above. My hypothesis is that you'll need to assume absolute physical properties described in isolation from other physical systems in order to complete any such derivation. This would still be an amazing derivation as it would presumably make clear that all other interpretations (whether psi-ontic or psi-epistemic) have a real problem / inconsistency with measurement as you say (and which I believe), but I'll bet Relational QM will be unscathed since it does not grant the assumption of absolute physical properties described in isolation from other physical systems ie, that the wave function of an isolated physical system is either real or some part of "knowledge" of something else. There is no such thing as an isolated physical system. Such a notion in Relational QM is physically meaningless.

    BTW, I read your previous blog post about Relational QM and I see that you do not rule it out or find an inconsistency in it other than to note that it makes you queezy to think of "reality" consisting of such conversations with your grandmother or that such is the case with Rovelli's dog :)

    ReplyDelete
  18. Sabine's post ends with: grab a pen and get started.

    I grabbed my pencil and tried to think of experiments which might help test various ideas about the measurement problem. I came up with, cold (low noise), very small detector, something like NISQ? And I asked about experiments (Are there any experiments, even in-principle experiments, which could help resolve the quantum measurement problem?)

    Sabine replied: "Yes, there are. Basically, if you reduce noise sufficiently much you should see correlations in measurement outcomes that, according to quantum mechanics, should not exist. This requires to minituarize the experiment as much as possible, keep it cool, and repeat measurements as quickly as possible."

    So far, no others who have posted comments have written anything about possible experiments that might help resolve the measurement problem.*

    What do you Jim, isometric, Lawrence Crowell, Chris, Charlie, Yehonatan Knoll, Andrei, jim_h, Gokul Gopisetti, Dr. A.M. Castaldo, Peter Shor, Enrico, tytung, Andrew Dabrowski, manyoso, Antonio (aka "un físico"), Udi Fuchs, Reimond, SG, dtvmcdonald, Luke, Alex, Q, Pmer, Murpheus, Florin - etc - think of Sabine's idea? Are there other kinds of experiments which might help resolve the measurement problem?

    * thanks for your comment B.F., but would you please try again?

    ReplyDelete
    Replies
    1. If KATRIN does not see a mass, then tiny Lorentz violations could explain neutrino oscillation. This is a hint that something like Penrose gravitational objective reduction could be the solution to the measurement problem as I wrote here.

      Delete
    2. JeanTate,

      The measurement problem exists only for those that assume QM to be a complete description of nature. I do not believe this. In my opinion the quantum state is an incomplete description of the system in a certain experimental context. Because the description is incomplete it is compatible with many experimental outcomes (it is under-determined). When the experimental results are available, we see what the outcome is. This is the "collapse".

      I find a theory like stochastic electrodynamics (classical EM + assumption of a primordial EM field that plays the role of the QED vacuum) the most promising path towards the completion of QM.

      Delete
    3. Thanks Andrei.

      So, what (new) experiments (even if only in principle) can you outline that would test stochastic electrodynamics, especially to distinguish between it and QM?

      Are there any such experiments that you can think of that would relate to the measurement problem (per the blogpost)?

      Delete
    4. JeanTate,

      Stochastic electrodynamics (SED) was not developed with the intention to be a QM interpretation. It was an attempt to "repair" classical electrodynamics. It did succeed in providing a classical explanation to problems like black-body radiation, specific heat, Van der Waals forces and recently, the nature of the electron's spin. It has failed to this date to reproduce the hydrogen spectrum but it still improved the atom stability problem. So the problem is not to distinguish it from QM but to find a way to completely reproduce QM. The problem is difficult because it requires a good understanding of the nature of the electron, which, to my knowledge, has not been accomplished even in QED.

      For a good introduction you can look at:

      Stochastic Electrodynamics: The Closest Classical Approximation to Quantum Theory, Atoms 2019, 7(1), 29

      https://www.mdpi.com/2218-2004/7/1/29/htm

      There is also a free book, "The Emerging Quantum". You can find it here:

      https://loloattractor.files.wordpress.com/2014/11/luis_de_la_pec3b1a_ana_marc3ada_cetto_andrea_valdc3a9bookzz-org.pdf

      This theory does not have any measurement problem. It is a strictly deterministic theory (that's confusing, given the name "stochastic"). Quantum probabilities are classical probabilities. The system is always in a single state, before the measurement and after the measurement.

      Delete
    5. Thanks Andrei.

      Unless and until "completely reproduce[s] QM" - or at least the experiments whose results are consistent with QM (especially, for this blogpost, the various "Bell tests") - SED is interesting but little more, right?

      Of course, "QM" has a lot more than atoms and electrons, the Standard Model (of particle physics, not cosmology) for example; how does SED fare wrt that?

      Delete
    6. JeanTate,

      SED is a work in progress, sure. It is a theory about EM interaction only. However, if it can come up with a good model for the electron I am quite sure that we will understand more about the other forces as well.

      What I like about it is that it does not imply any "weird" stuff. The world is just like it appears to be. No "many worlds", God-like entities like the ontic Psi, no retro causality, non-locality, etc.

      Delete
    7. In an experiment similar to Sabine's suggestion, you might attempt a double slit experiment in a highly shielded, near-0K environment, where the ambient electromagnetic radiation is minimized. If this resulted in a significant reduction in observed interference, then a natural, physical cause for quantum behavior would be established, mooting the measurement problem.

      Delete
    8. JeanTate,

      I'm not a scientist. My idea of the way forward is to stop trying to construct theories that exclude the conscious observer.

      Delete
  19. Jim, I am looking forward to your book. For me, Relational QM is the best interpretation I've seen. You can think of it as the most anti-realist interpretation at all. In fact, as I'm arguing with Sabine below I don't think it is fair to describe it as psi-epistemic at all.

    Relational QM goes further than all the so-called psi-epistemic interpretations by denying the physical existence of absolute independent systems. In other words, I would say that Relational QM would indeed deny that the moon exists when you are not looking at it. Why? Because the question presupposes the existence of absolute independent systems. The moon is not a real physical thing. The only real physical things are the relations or correlations between systems. The systems themselves (in isolation) are not physically meaningful.

    So my position is that Sabine is *right* that their really is a measurement problem and she describes it quite well. Moreover, I think every interpretation that assumes the existence of real absolute physical systems falls prey to it. If she can indeed derive this it would be amazing.

    ReplyDelete
  20. Note, that Gil Kalai thinks it impossible to reduce the noise sufficiently to achieve quantum supremacy and this might have implications for your proposed experiment?

    https://arxiv.org/pdf/1908.02499.pdf

    ReplyDelete
  21. In the preceding comments, the word "simple" or its derrivatives (simply, simplify") appear 34 times.

    And yet...

    ReplyDelete
  22. Jim, quote: My position is: "the world is quantum-mechanical and local" because nature respects locality, Lorentz invariance and is quantum mechanical.
    Then, please explain me the "spurious" correlations of Bell-measurements. If the "real" measurements are done at the position of the observers, the coincidences we find should obey the Bell limit. But they don't. So, there has to be some "spooky" exchange of information "at a distance". The argument "well, it's quantum..." doesn't help much in the case and Einsteins EPR-argument still looms.

    With Bohm-like determinism, it's not spooky longer. With Bohm, the hypercorrelation of the measurements is a spurious one, caused by specific initial postions of the two particles. In this sense, the Bohmian approach is strictly local.

    ReplyDelete
  23. Bee,

    With the problem definition, do you mean:

    In the event/setup chain of:

    carefully prepared wavefunction => measurement => amplification/transportation of the measurement result information => human laboratory operator watching the outcome

    The operator sees only one result per test run (and statistically distributed results in long run), thus being able to conclude the collapse of the wavefunction (which requires non-linear, non-unitary operation).

    And the problem being, that we don't know when or where or how the non-linearity/unitarity actually happens?

    -Topi

    ReplyDelete
  24. "What does it mean for us to live in a universe where Bell's inequalities are exceeded?"
    OK The Bell inequalities arise when you attempt to reproduce correlations that arise in quantum interactions with classical operations such as determination of the orientation of a physical particle.
    Classical systems that are sensitive to orientation of some object in space will always produce at best, a straight-line correlation curve between correlation and anticorreltion when a pair of unconnected analyzers are moved from aligned to 90 deg - simply because of circular symmetry. I did it for fun, using a computer to simulate how pairs of twirling batons would pass through a grille.
    Quantum experiments e.g. photons through analyzers show a Cos^2 theta correlation curve. The disturbing thing for me, is that people then jump in and assert that since these classical determinations of orientation can only produce a straight-line correlation curve, then quantum interactions require a non-local effect to produce a simple cos^(theta) curve. Just like 'collapse' there is no actual explicit description as to how this achieved.
    Non-locality is to me, just like collapse, simply an ad-hoc move to rescue the hypothesis that the 'particle' is the wave-function.
    There is a much simpler option: If the interaction with an analyzer cannot be reproduced by a measurement of the orientation of an object, then it could just as well be that the kind of information that results from the interaction between photon and analyzer does not determine the orientation of the incident particle.
    Rachel Garden: Int Journal Theoretical physics 1996.
    "At the end of the paper
    this analysis is used to refute arguments that quantum mechanics is nondistributive
    and that the failure of Bell's inequality in quantum theory threatens our conceptual
    scheme. Instead we reach a much less drastic interpretation of quantum mechanics."
    Simply put: A polarization analyzer is a device that when it is 'excited' by an incident photon, then it emits a new photon that is either aligned along one of two axes.
    If a photon emerges in channel 'A', all you can say is that its original state was not Exactly aligned with 'B', which is a denial, not a negation.
    The proof of the Bell inequality depends on taking the outcome of an interaction as providing classical bivalent information. If you use the actual logic, as Garden proves, then violation of the Bell inequality is not only possible, it is expected.
    So what is it like for us to live in a universe where Bell inequalities are exceeded" - absolutely fine, the only bad part is the hostility one seems to get when suggesting that those that dine out on lecturing the public that Einstein was wrong, ad that QM implies wooooo ... 'nonlocality', because 'Bell' might in fact be dreadfully mistaken.

    ReplyDelete
  25. Reimond,

    I'm probably just in over my head, but I don't see how the bubble paradox is a real problem for locality; maybe in 1909, but not after Feynman. Can't you describe the situation as virtual particles taking all possible routes from the original particle, each in its own superposition, in which energy is conserved locally. The collapse then consists in a selection of a particular superposition, but nothing discontinuous happens in it.

    ReplyDelete
  26. If we define an observer as an algorithm that processes information in a specified way (defining the algorithm then defines the identity of the observer), then that addresses the basis problem in the MWI. As I've pointed out here:

    http://fqxi.org/community/forum/topic/3319#post_151422

    imagining an observer in a classical setting leads to paradoxes that can be addressed within the MWI framework.

    ReplyDelete
  27. Peter, Andrew

    I think it is possible that the new theory that replaces QM will have the following property:
    a) does not have a measurement problem (i.e. it explains why it has definite measurement outcomes);
    b) reduces to QM and its predictions when we restrict our observations to QM measurements. Therefore it does not give different predictions for QM measurements.
    c) suggests new kinds of measurements (beyond QM measurements), and these are testable.

    ReplyDelete
  28. antooneo,

    The relevant quantity is what is called the wave-vector, which is proportional to the *four* momentum. It has a zero-component with, in the rest frame, is proportional to the mass. The wavelength can be interpreted as the inverse of that. Really this is textbook stuff, please look it up.

    ReplyDelete
  29. Hi Sabine,
    One of the issues I have with the discussion is we talk so much about 'measurement', which of course leads to many misinterpretations that start involving human consciousness etc etc... but what does measurement truly mean? Removing the human implications of the word 'measurement', I have read (I think long ago in Greene's Elegant Universe), reduces the definition to 'thermal disturbance' with relation to quantum systems. As far as my relatively layman brain can tell, this further reduces the definition of 'measurement' to [any process by which information is exchanged between fields or their particular excitations].
    This leads me to ask; what does wavefunction collapse and gravity look like when viewed through the lense of Verlinde's entropic gravity / information-theory approach?

    ReplyDelete
  30. Jim,

    I'm not a realist, but to state the obvious, reductionism has worked very well. Reductionism is not a philosophy, it's an empirically supported fact that things are made of smaller things.

    If you are advocating neo-Copenhagen Psi-epistemic interpretations, you are asking me to ignore 2000+ years of insights into the workings of nature.

    Really supporting neo-Copenhagen approaches is as inconsistent as supporting strong emergence or talking about top-down causation. There just isn't any such thing. No one has ever seen anything like that. We do not even have a theory for it. It's just bla-bla. Sorry if I can't take this seriously.

    ReplyDelete
  31. @Andrei When you do a measurement in an EPR experiment you update your knowledge about the system. Before the measurement (and after the preparation) you know that it is in a certain state, say (|↑↓⟩−|↓↑⟩)/√2. After the measurement you know it is |↑↓⟩ or |↓↑⟩ with certainty.
    The state vector is independent of position, so it doesn't matter where the particles are when you do the measurement and as Murray Gell-Mann said, talking about "local" or "no-local" in this context is like "giving a dog a bad name" (https://www.youtube.com/watch?v=gNAw-xXCcM8).


    ReplyDelete
  32. Sabine,

    Okay, but now I'm really confused. You say you're not a realist, but a realist about what? Objective reality? Surely not. Entity realism. I would guess not. The realism of the quantum representation? But this would make you an anti-realist according to the definition I'm using.

    And reductionism IS a philosophical position. You'll find these definitions in the Oxford Companion to Philosophy:

    Ontological reductionism: a belief that the whole of reality consists of a minimal number of parts.

    Methodological reductionism: the scientific attempt to provide explanation in terms of ever smaller entities.

    Theory reductionism: the suggestion that a newer theory does not replace or absorb an older one, but reduces it to more basic terms. Theory reduction itself is divisible into three parts: translation, derivation and explanation.

    Yes, reductionism has been the basis of science for a very long time, and you're right to claim empirical success. But there is no empirical evidence to support the conjecture that reductionism is guaranteed to work ad infinitum. Anti-realist interpretations do indeed suggest that in quantum mechanics we've hit a fundamental limit in terms of what is accessible to empirical science, but you're perfectly at liberty to ASSUME (without proof) that there's a deeper layer that is empirically accessible but which has yet to be discovered.

    My confusion arises because this is typically the assumption of realists such as Lee Smolin. And yet you said ...

    ReplyDelete
  33. Jim,

    I know that philosophers try to claim reductionism as a philosophy. As I have pointed out, however, it's established fact that nature is reductionist. It is right that we have no way of know that this will continue to be so, but if you are claiming this is not the case -- as ne-Cophenhagen approaches do -- then you are making a scientific statement, not a philosophical one. You will have to explain me where and why reductionism stops working. I want to see a theory for that, not just a shoulder shrug.

    I'm an instrumentalist all the way down and don't subscribe to any form of realism. I merely point this out because you seem to be accusing me of being a realist when really I am just pointing out that neo-Copenhagen approaches are axiomatically unsatisfactory.

    ReplyDelete
  34. Andrei (10:48 AM, August 15, 2019) asked,

    "Can you please describe what do you think happens in an EPR experiment without any reference to hidden variables? What is your local explanation of the observed results?"

    Thank you. The question that is avoided being answered.

    Maybe time to forget about Hilbert spaces, wave functions, path integrals, many worlds, etc. and do something else.

    ReplyDelete
  35. Sabine,

    you wrote: "I am just pointing out that neo-Copenhagen approaches are axiomatically unsatisfactory"

    Who said that science should be “satisfactory”?
    Some find very “unsatisfactory” that the earth is a sphere and that species evolve with time.

    But ok, let’s put ourselves in this dissatisfaction mood and agree with you that the Copenhagen interpretation is “unsatisfactory”. How can we fix this?

    Hidden Variables:
    Ruled out systematically since 1972 (and specially since1982) by multiple experiments.

    Superluminal action:
    Not compatible with locality, causality, Lorentz invariance, SR.

    De Broglie-Bohm pilot wave:
    Let’s build an apparatus to measure this field. I personaly don’t know what is suppose to be measured, but it’s ok may be some vacuum of some sort.

    What is your bet ?

    ReplyDelete
  36. MarkusM,

    This is not an explanation. An explanation should present a logical link between the preparation of the experiment and the experimental outcome. You take the quantum state as representing knowledge. So, what you are saying is that given a certain preparation procedure you know how the results would be like. This tell me nothing about nature. If you press the "4" button in an elevator you know that the outcome will be the arrival at the 4th level of the building. Do you think this is an explanation of how an elevator works?

    ReplyDelete
  37. isometric,

    What I mean by "axiomatically unsatisfactory" is that they do not solve the measurement problem, which is a problem that requires solution, as I have explained in my blogpost. Neo-Cophenhagen approaches postulate that reductionism breaks down somehow but do not explain when or where or why. This isn't even a theory, it's just words.

    My bet, needless to say, is superdeterminism. (Which you have already forgotten about again.)

    ReplyDelete
  38. Looking at fractions differently. Let us consider the fraction 1/1. Here, we are asking what is 1 with reference to 1? It is one and the same thing. Let the numerator be a thing, and let the denominator be the reference or the frame of reference. Now what is 1 with reference to 2? 1 is halved or 1 looks like it has grown smaller in magnitude when we enlarge the frame of reference or alter the frame of reference; its numerical definition changes. Suppose we decrease the frame of reference to ½, then we have 1/half. Then 1 looks like it has grown to twice its original magnitude with reference to half. If I further decrease the frame of reference to (one fourth), then we have 1/ (one fourth). Then 1 will look like it has grown to 4 times its original magnitude. Let us call this process “defining” the magnitude of a thing “1”. We keep on repeating this process until the frame of reference i.e., the denominator approaches 0. Then when the frame of reference is 0 what is “1”? “The undefined”. The frame of reference has vanished which is akin to removing the observer, and I can’t numerically define the thing “1” anymore.
    Conversely, if I keep increasing the frame of reference i.e., the denominator, the numerical definition of the thing “1” changes or decreases. When the frame of reference becomes infinitely large, then definition of the thing “1” becomes insignificant; it loses its individuality, its separateness. Consequently, there can be no comparison with the frame of reference, therefore measurement stops, the thing “1” merges into infinity and becomes one whole thing. Because measurement stops, because there can be no measurement, there is no frame of reference, there is no observer, which is signified by 0. 1/(infinity) = 0.

    ReplyDelete
  39. Sorry, this is not correct. The wave-length is lambda = h/p where p is the momentum.

    ReplyDelete
  40. manyoso,

    "The only *things* that *exist* and have physical meaning are the relations. The relations are not "knowledge" or "information" about independent physical systems, but rather the relations *are* the physical items themself."

    The word "relation" is meaningless in the absence of the objects between each the relation is supposed to exist.

    ReplyDelete
  41. my last post continued...
    Comparison can take place only when there are at least two thing. When there is one whole thing, then there is no scope for comparison. There can be no measurement without comparison. In this context, 0 can also mean the point where measurement stops;the point where measurement breaks down.

    ReplyDelete
  42. antooneo,

    You clearly do not understand what I am saying. Please look it up in a textbook.

    ReplyDelete
  43. I think I see the source of the confusion. You want to clearly distinguish 'science' from 'philosophy', and you have some very clear notions about what science is.

    But I (and many others) argue that you can't do science of any kind without philosophy, in two senses. All scientific theorising requires some metaphysics, in the sense of assuming some things we can't provide empirical evidence for. This is actually okay, provided that we can establish a tension between the resulting theory and the empirical facts. In theories where the metaphysics becomes overwhelming and there are no empirical facts, and hence no tension, then we're firmly into post-empirical science.

    The second sense is concerned with the *methodology* of science. Anything to do with methodology is by definition metaphysical - you can't provide empirical proof that one or another 'scientific method' is correct, which is why there's no consensus on this. All you have is judgement, based on your experience as a scientist. For many years the scientific method was understood to be based on induction and verification. Then falsifiability. Then any one of a number of different approaches. In this sense, I consider reductionism as part of the methodology. You ASSUME that nature is reductionist because this assumption has paid off in the past. There's absolutely nothing wrong with this judgement, but you can't claim nature is reductionist as an 'established fact' because you have no guarantee that this assumption will continue to pay off in the future.

    Anti-realist neo-Copenhagen interpretations don't make scientific statements, as you claim, they make philosophical statements. The only scientific statements that can be made about QM concern the success of the mathematical *formalism* (state vectors in Hilbert space) in the light of empirical data, and these data don't currently shed any light at all on how the formalism should be interpreted. Before you cry 'But … Bell's inequality', remember that the Bell and Leggett-type tests are based on realist local and crypto non-local hidden variable EXTENSIONS of QM. What these tests show is that the inscrutable formalism - without extensions - works best.

    I don't expect you to buy all or any of this, though I'm curious to know what you mean by 'axiomatically unsatisfactory'.

    ReplyDelete
  44. Sabine,

    Thank you for your clear answer: "superdeteminism".

    Super determinism is still a mystery to me. Is it the perfect knowledge of initial conditions up to the 10^35 decimal and the exact solving of differential equations (without using perturbation or numerical methods)? It seems very very very ambitious.

    As long as QM in its actual concise formulation reproduces experimental results correctly the Copenhagen (psi-epistemic, probabilistic, local) interpretation is ok. No need for a cure.

    ReplyDelete
  45. Jim,

    I have literally written a book about how you cannot do science without metaphysics, but thanks for explaining this to me.

    I am fully aware that science ultimately rests on philosophical principles. This doesn't negate the fact that some problems can, and historically have been, solved by certain procedures. Whether you call those scientific or not, I'll leave up to you - the terminology absolutely doesn't matter. I am saying if it's been successful before, the smart thing to do is try it again. Reductionism has been tremendously successful. So has been resolving axiomatic inconsistencies. We should do the same thing with the foundations of quantum mechanics.

    Neo-Copenhagen approaches are merely attempts to hide an axiomatic problem behind a fog of philosophical mumble. No one learns anything from that. Having said this, it's not like I have a problem with people who believe it. I am just saying it's not a promising route forward.

    ReplyDelete
  46. manyoso

    The relational interpretation is firmly in the psi-epistemic camp. Here, Rovelli is quite emphatic about it.

    ReplyDelete
  47. Gokul Gopisetti: I'll disagree. You are treating (infinity) like a number, which it is not. We can compare 1 to 7, or 0.5, or 50,000,000 because these are actually numbers. Infinity is not a number, it is a concept. If (1/0=infinity, then 1=0*infinity, but 0*anything = 0, so 1=0, a contradiction, so 1/0 does NOT equal infinity. Nor does 1/inifinity = 0, for the same reason. Infinity is not a number, at best it is a concept for an arbitrarily large number, one as large as you care to make it. Likewise, zero is a place holder for "nothing", and we must always be careful when working with that concept.

    And if you wish to compare numbers, you must compare them to numbers, not concepts outside of the number system. You might as well end your discussion by comparing 1 to "romance" or some other concept.

    To speak correctly, 1/infinity means comparing one to any arbitrarily large number, and the limit of 1/N as N->infinity is indeed zero, but that doesn't mean 1/N for any number N is going to be zero; just that it can be made as close to zero (without being zero) as we wish it.

    Then, if there are physical constraints not represented in the equations (for example if there is a lower limit to mass or energy or distance in space due to quantization) then even this is no longer true, we cannot get two particles to approach arbitrarily closely, e.g. there would actually be a physical lower limit of closeness, or perhaps a physical upper limit to the curvature of space, etc.

    These would violate the concept of infinity, that a number can be arbitrarily large, or 1/infinity, that a number can be arbitrarily close to zero. i.e. those limits may be physically impossible to realize.

    ReplyDelete
  48. Sabine,

    I don't think it's philosophical mumble (why are you so dismissive?), but I'd actually go further. The anti-realist interpretations are passive - they don't provide ANY route forward. All the exquisite experimentation on entangled quantum particles was/is motivated almost exclusively by realist alternatives. And the fact that the standard formalism has triumphed every time has led me to a general feeling of uneasiness …

    Good luck with finding the source of the axiomatic problem. The current axioms of QM - completeness, Hermitian operators, expectation values, the Born rule, the unitary time-dependent Schrodinger equation - were all born under the influence of Copenhagen. Lucien Hardy sought to rewrite these as a general form of probability theory, but the result is yet another anti-realist interpretation.

    ReplyDelete
  49. Jim,

    I am not "dismissive", I have thought about this back and forth for a long time, and you fail to even acknowledge my point. Neo-Copenhagen interpretations postulate the failure of reductionism but do not put forward any explanation for how and under which circumstances this is supposed to happen. Unless someone provides an explanation for how this is supposed to work, I will continue to call it philosophical mumble, just like ideas about strong emergence and top-down causation and holistic whatevers.

    I have already told you what the axiomatic problem is. Any reference to "knowledge" or updates of that knowledge (or decision theory and such) implicitly promotes such concepts to fundamentals, when reductionism tells you they should be emergent. In that case you have to tell me where the break is supposed to happen. Where is the theory?

    I am not proposing an anti-realist interpretation. In fact, I am proposing a realist interpretation. I am merely pointing out that I am not doing this due to some philosophical preference, but simply because it serves to solve a problem.

    ReplyDelete
  50. Sabine,

    There is no "measurement problem" in QM. Your premise is wrong. The double slits experiment, the singlet state, macro-molecules scattering, etc. are well explained in the QM formalism (+ the Copenhagen interpretation if you need a metaphysical layer upon it).

    Nature is (as far as we know without new experiments):

    Quantum mechanical and local.

    You are trying to find a super-mega solution (“superdeterminism”) to a non-existing problem.
    You are trying to repair a car which runs fine with a big big gigantic hammer.


    ReplyDelete
  51. Sabine,

    And there I was actually agreeing with you …

    The basis for the anti-realist assertion that further reduction might be impossible is based on a long history of philosophy dating back at least to Democritus. Yes, it's a philosophical argument - don't expect an equivalent scientific assertion until we've spent another few hundred years trying to discover empirical evidence for a deeper level of reality.

    And yes, the track record of empiricists or anti-realists isn't great - Mach, to pick one example, got atoms hopelessly wrong.

    I'm still not sure I understand your hangup about 'knowledge'. Anti-realist intepretations such as Rovelli's relational QM don't make any big deal about knowledge and certainly doesn't elevate it to a fundamental concept. Yes, it's based on the notion that the wavefunction is simply a way of 'coding' our experience of the quantum physics, but you can just as easily replace 'experience' with 'all the empirical facts'.

    I'm still confused, I'm afraid. At 4.48 AM you posted 'I'm an instrumentalist all the way down and don't subscribe to any form of realism'. Now, at 7.16 AM you post 'In fact, I am proposing a realist interpretation'.

    I don't propose to continue this discussion here, but am looking forward to talking when you're in Oxford next month.

    ReplyDelete
  52. isometric,

    I have explained exactly what the problem is in my blogpost. If you want to deny the conclusion you have to find a mistake in my argument, not come here to proclaim you don't like what I say.

    ReplyDelete
  53. Sabine

    As I suggested below, I think you need to defend your implicit claim that reductionism itself demands that a theory - and its interpretation - be [reducible to a] representational [one]. Otherwise, your position certainly is due to a philosophical (metaphysical) preference.

    For those of us who don't share that metaphysical preference, the "sentiment that quantum physics is [applied] quantum probability theory" (and that its irreducibly probabilistic nature isn't a problem needing a solution) is very well motivated.

    ReplyDelete
  54. Paul,

    I don't know what you mean by "representational", sorry.

    ReplyDelete
  55. Sabine

    The meaning of "representational" is given in Bub's 'Two Dogmas' Redux paper which I linked to in my earlier reply to you.

    ReplyDelete
  56. manyoso,

    I've looked at Rovelli's interpretation a little bit and I noticed that it is based on the assumption that macroscopic superpositions (Schrodinger's cat states) exist. I think I have good evidence that such states cannot in fact exist. The states of a macroscopic object, if different enough (like different pointer positions) produce observable consequences in the entire universe as a result of gravitational, electric or magnetic fields associated with their state. That means that once the superposition goes beyond the uncertainty imposed by Heisenberg's principle it "collapses" for all observers everywhere.

    ReplyDelete
  57. What kind of correlations do you hope to see? It is very difficult to add any kind of extra prediction to things that are supposed to be random in standard quantum mechanics without letting parties communicate faster than light.

    ReplyDelete
  58. Jim,
    there must be a misconception here. There is no "prefered" basis,
    except perhaps for the theoretician who finds angular momentum
    eigenstates more convenient.
    For the detectors, what arrives is just unpolarized light - it could
    just as well be a mixture of horizontal and vertical photons.
    There is no fact of the matter whether the state is "really"
    circular or linear, or elliptical. Having a ket is not enough -
    it must always be combined with a bra and be integrated over
    to arrive at a physical result.

    Werner

    ReplyDelete
  59. as a complete lay person: on the double slit experiment. One foton as a wave travels through both slits to a screen. The wave hits the screen in an interference pattern.

    Could the chain of events not be that the foton/wave cause one electron or atom of the screen to absorp its energy, and that the chances of the location of that "hit" is reflected in the interference pattern. The "collaps" of the wave is then caused by the absorption of its energy.

    Most likely I am talking nonsense.... but I like to know why?

    ReplyDelete
    Replies
    1. In this experiment - of which there are many variants - the photon (or electron, or buckyball, or ...) is a quantum thingie. The slits and screen are not (they are classical thingies).

      What if the slits and/or screen were quantum thingies instead?

      Do you think it possible to create a double slit experiment in which either the slits or the screen (or both!) are quantum thingies?

      Delete
  60. isometric wrote: "Do not reverse the roles. You are in charge of proposing an experiment which is not compatible with QM and the Copenhagen interpretation (call it QBism or Psi-ontic if you like fancy terms). [...] As long as something like that is not presented, I’ll stick to the interpretation of the wave function as a complex distribution function."

    Thanks. I want to be clear, are you saying that the measurement problem (per this blogpost) does not exist?

    ReplyDelete
  61. Thanks for your slightly longer response, B.F.

    May I take it, from this response, that you think there are no new experiments which could be done (even if only in principle) that will address the measurement problem (per the blogpost)? That the only discussions to be had are philosophical/theoretical?

    OTOH, if - as "We actually care about knowing more about this, be it merely theoretical, or much more preferably, experimental" implies - experiment does have a role to play, what *new* experiments can you outline that might be relevant to this very lengthy discussion?

    ReplyDelete
  62. Somehow my comment didn't appear in the last two days. Let me try in more compact form, removing a link to Nature magazine. We now know that:
    Sub Quantum Mechanics Exists
    Indeed, we now know that there is another process, deeper than Quantum Mechanics at work: Michel Devoret and Al. were able to predict Quantum Jumps (published in June 2019 in Nature journal). Quantum Mechanics is thus incomplete: it can’t predict itself (but Devoret can!)
    Call this existing situation Sub Quantum Physical Reality (SQPR).

    The question with SQPR is what could it be? Nonlinear, for sure, because so are the “jumps”.
    Copenhagen interpreters may say that this brings nothing: how do they know? This is no hypothetical debate in hermeneutics, but finding out what is really going on (defining “really” in the process). SQPR could well bring predictions (or postdictions?) such as, for example... Dark Matter. At least, so I believe.

    Notice also that local time (central to Poincaré’s Relativity theory, as it enables to deduce the rest of Relativity) is a Quantum phenomenon… Because it counts the impacts of photons oscillating between mirrors. Thus SQPR is not bound by Relativity (thus to claim SQPR violates Relativity is no objection).

    Patrice Ayme


    ReplyDelete
    Replies
    1. Patrice Ayme,

      Can you please briefly summarize how SQPR intersects with the measurement problem (per this blobpost)?

      Delete
  63. Martien,

    Why do you think the foton travels through both slits?

    ReplyDelete
    Replies
    1. Because it is (also) a wave

      Delete
    2. Martien,

      How do you know it is a wave?

      Delete
    3. From the reference pattern resulting from the double slit experiment it was deduced that whatever a photon is, that it has at least under certain conditions wavelike properties.

      Delete
    4. We know that waves always produce interference patterns. The correct logical implication is that in the absence of an interference pattern you do not have a wave. It does not follow, however, that an interference-like pattern can only be produced by a wave. A particle subjected to a suitable field will assume any trajectory you like (like the electrons in TV tube), including trajectories that will generate interference-like patterns.

      Delete
  64. Angular momentum is a quality that can be carried by a moving constituent of energy without representing the directional momentum of the particle. Angular momentum changes between two perpendicular dimensions as a sinusoidal modulation.

    This is one of the more perceptive comments here, in as much as, it engages the fundamental question of where mathematical and physical descriptions might converge to give a fully scientific account of quantum behavior. This analysis suggests that the Bell tests do not demonstrate a divergence between classical and quantum expectations at all. Rather, it demonstrates an analytical failure to properly calculate the classical expectation in the first place.

    The situation then is somewhat akin to the Keplerian expectation error with regard to galactic dynamics. When there is an unrecognized qualitative or quantitative error in determining expectations, the resultant discrepancy can be attributed, conveniently, to an externality such as dark matter or entanglement, leaving the fundamental error unacknowledged.

    ReplyDelete
  65. The difference is that the classical goat was behind the door before its detection there, whereas in the quantum realm the formalism is widely interpreted as implying that the goat is superposed between all three doors before detection. If superposition is the case, then you have a measurement problem. If the quantum goat is always behind one door then you have a local hidden variable problem, since the Bell tests seem to preclude that possibility. Both problems may just be dependent on the inarguable incompleteness of the wavefunction description (inarguable, unless you're a Copenhagenist of one form or another).

    ReplyDelete
  66. B.F,

    We are totally happy if you can give us some evidence for the breakdown of reductionism.

    The Big Bang Theory (pejorative usage intentional).

    ReplyDelete
  67. Martien: you are not talking nonsense.

    The photon is NOT absorbed by just one atom, in a typical solid. It certainly can be in a gas.

    The electrons in a solid, or even a single molecule, are spread over more than one atom. In a surface like copper, they are spread over a very lot of atoms.

    But after a measurement, it will be found that the photon was, usually, in an area much smaller than the spread of the photon as it approaches the screen. The cause of that is a big part of what is being argued over here.

    ReplyDelete
    Replies
    1. dtvmcdonald

      Thanks for clarfying this. The point I wanted to make is this. If you shoot photons one by one through a double slit, they hit the detection screen one by one and gradually the interference pattern appears. This patterns is - if I am correct - attributed to the photon/wave which collapse, which process is not well understood or not understood at all.

      What id the appearing pattern is not to be attributed to the incoming weave, but to the detection screen? At one "point" the screen absorbs the energy of the incoming wave. (Whether that is one atom or more atoms doesn't really matter.) Once the ewnergy is absorbed the photon/wave is gone ('collapsed').

      Id the chances of where the screen absorbs the incoming photon is proportionate to the energy of the incoming wave, we get the same interference pattern.

      But, alah, this is just a thought-experiment, and I can imagine that t here are many experiments done, which show that my reasoning is flawed. But if so, I would appreciate4 somebody to explain to me why. Thanks

      Delete
    2. To clkarify further. The 'collaps' is often discussed as a property of a wavefunction, happening in interaction with its environment. Isn't it possible that the collapse is to be seen as a property of the wave plus its environment?

      Image again that single photon coming from the double slit approaching the detection screen as a wave (with the wellknown interference pattern).

      Image the detection screen as zillions of atoms lined up. At the moment the wave arrives its energy is absorbed in one place on the screen. The possibility for this place to be in a certain spot in proportionate to the energy density of the wave, and is further determined by the detection level. Maybe random quantum fluctuations.

      Alas, this probably wouldn't work, because in this thought-experiment, time/distance matters, and the place on the screen which is hit first would be the 'winning spot'.

      Nevertheless it looks common sense to me to see 'collaps' not only as a property of a wave, but as a property (or better the result of interaction) of a wave plus its environment.

      In some case a collaps could be equal to emergy absorbtion?

      Best,

      Martien

      Delete
    3. In which case the collapse of a wave function is the transformation of one type of energy in another type of energy in a physical system. It then can only be meaningful understood on system level.

      Delete
  68. JeanTate,

    Yes, I think that the "measurement problem" does not exist and is therefore a bad starting point for further research in physics.

    It's eventually a metaphysical problem or a dissatisfaction but not a physical problem. But it's ok if people want to think about that.








    ReplyDelete
  69. Sabine: yes, NISQ is different from what you outline. However, what I might call NISQ technology might give an experienced experimenter some pointers, e.g. how to reduce (or accurately characterize) noise, how to build tiny detectors.

    manyso: Kalai's paper provides hints for someone looking to design an experiment sorta like what Sabine outlined. I think the "why" of an inability to achieve quantum supremacy is related - possibly closely - with the measurement problem (per this blogpost).

    Peter Shor: isn't it more to do with doing an experiment to find out, rather than nailing down what kind of correlations one hopes to see? Perhaps one way to view this is testing QM in a physical regime where no tests have been done up till now ... "quantum scale detectors" perhaps.

    ReplyDelete
  70. I don’t normally react to People Being Wrong on the Internet, but dismissing the gap between the Higgs and Planck masses as a nothing-to-see-here, and suggesting in the same post to “grab a pen” and tackle something that was understood over 90 years years ago is too much of a trigger.

    If you let me, I’d like to say a few thing on why
    -what you call the psi-epistemic interpretation (i.e. plain quantum mechanics) has at most the same issues with “knowledge” (at a given time) being emergent as classical mechanics, and possibly less

    and why this implies that

    -your criticism, relative to the Schroedinger equation only applying for a limited period of time, is not even wrong

    Before learning about the character limit on posts, I wrote a text that was around 50% too long, so have to restructure a bit…

    The second point is in a way less controversial, I hope, once the first point is granted

    The Schroedinger equation is just a component of the algorithm that transforms this “knowledge” into odds. It even misses any reference to the final state!

    That it only applies in between poking events - that modify the system’s hamiltonian - is not a flaw or something unpredicted (lol!), it is how the machinery is supposed to work!

    So, let's move to this emergent “knowledge at one time”

    We need to assume at least one system, most likely an organism evolved through natural selection over millions of years, that is aware of the passing of time, able to receive and process information retain (or so it seems to it), and able to understand the concept of a differential equation. :)

    To this system, the initial conditions of a differential equation that is supposed to describe a physical phenomenon, always mean “assume you know”: there is obviously no need to “actually know” to use the equation.

    What is however needed, in order to “bridge the gap” between the mathematical model and the actual Universe, is the identification of a maximal set of mutually exclusive options that is compatible with what is “actually known”: this enables the use of confidence levels and of the rules of (classical, Bayesian) statistical reasoning, which he has figured out first by looking things at his scale, and then by filling the space around him with other systems in “known” configurations in order to extrapolate the “knowledge” at different scales.

    This is all is needed. And it is equally available in quantum mechanics as it is in classical mechanics, is it not?

    The *only* differentiator between classical mechanics and quantum mechanics is this specific limitation on how much can be assumed to be “knowable at a given time” about a given system in any given circumstance: in CM the maximal amount of “knowledge at one time” is also complete and in a way unique, while in QM it is *crucially* neither, which implies indeterminism.

    Although I’d love to write a few words about all the misconceptions that that keep circulating like headless chickens despite decades of royal debunking, both empirical and theoretical, I am approaching the limit length, so I’ll wrap it up.

    The discoverers of quantum mechanics, its linear structure, and its interpretation, the same people you seem too think were too lazy to come up with something better, stand right next to folks such as Newton, Maxwell and Einstein in terms of achievements of the human intellect.

    You see, even without possessing one of these minds, some people actually understand these things. They know why and how the conceptual structure came up as it is, and why it is inevitable.

    They may even answer to specific questions but beware that, based on my personal experience, they also find your remark about not worrying about this confusion you - and too many other who should know better - have, because “so they were told as students” particularly irritating.

    Cheers,
    Andrea

    ReplyDelete
    Replies
    1. ...in CM the maximal amount of “knowledge at one time” is also complete and in a way unique, while in QM it is *crucially* neither, which implies indeterminism.

      Yes, QM is incomplete with regard to the underlying physical processes that result in outcomes the QM formalism only statistically predicts. The question being debated here is whether that incompleteness of the model describes the underlying physical state or is merely an artifact of the formalism.

      The only argument you seem to have is that clever people like yourself know that the formalism is complete in its incompleteness. Self-referential arguments from authority don't really carry much scientific weight, especially in service of a circular argument. Cheers!

      Delete
    2. (eyes rollling)
      Your words are very confused, but I'll try anyway...

      The incompleteness of the model reflects the physical impossibility to do certain things. Thus there is no underlying "physical state" in the same sense as was imaginable had the Universe being classical. It has no constructible definition.

      On the second sentence, the argument I did make is that the formalism is as self consistent as that of classical mechanics... the only self-referential thing here is your delusion of understanding what you are talking about.

      Delete
    3. Andrea wrote: "I don't normally react ..."

      Nor do I. (Not normally.)

      "something that was understood over 90 years ago"

      Some theories have long gestation periods.
      For Maxwell it was obvious that the ether exists -
      how else could light propagate? (Obvious in a much
      stronger sense than QM seems to obvious to you.)
      Yet in 1905 it was discovered that the ether is not needed.
      And wouldn't you agree that Einstein's and Minkowski's
      "adjustments" made electrodynamics a much better
      theory? QM today may be in a state similar to that of
      electrodynamics before 1905, highly successful, but
      hampered by classical leftovers.

      "They know why and how the conceptual structure came
      up as it is, and why it is inevitable."

      This is called dogma.

      R.P.Feynman:
      I cannot define the real problem,
      therefore I suspect there's no real problem,
      but I'm not sure there's no real problem.

      Delete
    4. It has no constructible definition.

      Sounds like a math problem to me, but math isn't physics. In a physics experiment when an observable initial condition result in an observable physical outcome it is assumed that the outcome was a consequence of some physical process - even if that physical process is unobservable by typical classical observational techniques.

      Your position simply fetishizes ignorance of that unobserved process as a feature of your model. Science doesn't succeed by declaring its ignorance as fundamental and walking away from a problem. Thanks for your input though; now back to the mathematics department.

      Delete
    5. - bud rap wrote -
      Sounds like a math problem to me
      --

      No, dear confused stranger, I am of course referring to *physical* unconstructibility.

      In a classical Universe, it makes sense to talk about points in a phase space, and to use these things to label the possible configurations of a system (which is what they are used for), *because* you can assume the existence of a (limiting) physical process that “prepares” the system in each of those configurations. The mathematical concept of a phase space point is thus physically constructible, in a classical Universe.

      In OUR Universe, you may still *think* of points in a phase space, but the corresponding physical process turns out to be undoable. Physically impossible. Just-can’t-happen (well, unless the establishment as a fact of the uncertainty relations is put into question, which I put in the same category as questioning the relativistic speed limit).

      The concept of a phase space point, i.e. what I guess most of you confused people have in mind when whining about the “underlying physical state”, is NOT physically constructible.

      This means that the algorithm that turns “knowledge” into odds has to be built out of things that are NOT points in a phase space. Not because of dogma, to reply to confused stranger #2 - who might want to check his dictionary too - but because of *empirical evidence*

      Is there anything of what I’m saying that is getting through?

      Delete
    6. No. You seem to have slipped into a parallel universe. In this universe the link between your pronouncements and *empirical evidence* is broken.

      Delete
    7. Sure Werner, and since I guess this makes you rubber and me glue, feel free to go back to your affairs.

      Sabine, is there anything specific you don’t agree with, among the things I wrote so far? Any assumptions or implications that you think are unwarranted/irrelevant/unintelligible and thus invalidate the conclusion that your entire post should be retracted?

      To make things easier, I'll be satisfied with a reason why "actual knowledge" being emergent (which it is) should be more of an issue for a quantum fundamental theory of Nature than it is for a classical one, when a fundamental theory of Nature is, I'd say by definition, but certainly in practice, a tool that transforms "ideal knowledge" (and through bayesian reasoning also "actual knowledge") into odds.

      You (Sabine, still talking to *you*) can moderate what follows out, if you want.

      You’re not the only one, unfortunately, spreading utter misconceptions around, but you are the only one, so far, that is so smug to be personally insulting to other people (“because so they have been told as students”).

      So, assuming you are in good faith and not just full of it - riling your misguided fanbase up to sell books -, do you have the guts to back your claims in a rational “debate”?

      Make the rules, if just replying here does not suit you.

      Delete
    8. Andrea: as for personal insults, I think the score is much higher in your postings ("People Being Wrong on the Internet", "confused", "delusion", "smug").

      This is about quantum measurements. If you can't even perceive the problem ("Shut up and calculate!") feel free to go back to your affairs.

      Delete
    9. Werner, I wrote a few tens of lines, not a 100 pages treatise, containing a pretty specific argument that the “problem” is only in your heads.

      Yet, you have only been able to come up with (pathetically misunderstood, in some case) slogans and a sentence equivalent to “there is a problem”...

      For someone who knows what (s)he’s talking about it should not be that difficult, if the “problem” is really there, to tell me where I am so wrong.

      I repeat, as an alternative, I'll be satisfied with the reason why (actual) "knowledge" being emergent is a problem for quantum mechanics but would have been fine with classical mechanics.

      I have the impression I will be waiting indefinitely, but I'd love to be surprised

      Delete
    10. Andrea,

      I agree with you that by treating the quantum state as knowledge the measurement problem disappears. This comes however at the price of accepting a severe reduction of explanatory power. Let me clarify this by referring a simplified version of the EPR-Bohm test.

      We prepare a spin entangled particle pair (say by the dissociation of a diatomic molecule) and we measure the spin of the particles on Z. We will always get anticorrelated results, right?

      A realist interpretation of the quantum state (such as Bohm, or MWI) may refer to the quantum state itself to provide that explanation, because the quantum state represents the world.
      An epistemic interpretation cannot use the state in this way, because the state just reflects your expectancy about what is going to happen and there is no logical reason for nature to comply with your expectancy. So, nature must be described by some yet-unknown laws and those laws are in fact responsible for the observed experimental results. Q-bists for example openly admit that QM is not a theory of nature, but a theory about how to place bets.

      Now, you can accept that or you can try to find how nature works (so-called hidden variable theories). I think the second option better reflects the scientific spirit.

      Delete
    11. - andrei wrote -

      I agree with you that by treating the quantum state as knowledge the measurement problem disappears. This comes however at the price of accepting a severe reduction of explanatory power. Let me clarify this by referring a simplified version of the EPR-Bohm test

      ...

      Now, you can accept that or you can try to find how nature works (so-called hidden variable theories). I think the second option better reflects the scientific spirit

      --

      Hi Andrei,

      Not sure I get what you mean with "reduction of explanatory power". The explanatory power of QM is at least the same as that of CM: the decay products behave as they do because of

      1-their common “source”

      2-the conservation laws implied by the dynamics

      Literally the same words apply to both frameworks. Honestly can't imagine what more explanation of what is observed you would need.

      You may certainly complain about the lesser *predictive* power implied by QM´s indeterminism (though in some cases you won’t even have that! The indeterminism of QM is of a very, very special kind), but that complaint has unfortunately as much merit as complaining about the relativistic speed limit or the 2nd principle of thermodynamics...

      This brings me to your last sentence above, which calls for a huge, titanic, Nope.

      The scientific spirit, as you call it, definitely does NOT entail postulating entities/properties that are unobservable in principle!

      With an aphorism, how Nature works has to be inferred by what Nature shows: pinning down the internal workings of Nature with unnecessary assumptions - “has to be deterministic!” - is the *exact opposite* of a scientific attitude.

      Cheers

      Delete
    12. Andrea,

      "the decay products behave as they do because of

      1-their common “source”

      2-the conservation laws implied by the dynamics"

      Can you be more specific about the dynamics? What are the objects this dynamics is about? What part of the QM formalism reflects this dynamics?

      "The scientific spirit, as you call it, definitely does NOT entail postulating entities/properties that are unobservable in principle!"

      I did not say that you need to postulate such entities. However, they might be required in order to make a theory logically consistent and improve its explanatory power.

      It is also the case that what is observable and what is not observable depends on the theory you have.

      Electric, magnetic or gravitational (space-time curvature) fields are not directly observable but they are still very useful and assuming their existence improves the explanatory power of the corresponding theory. Hidden variables might play an equivalent role in QM. I think that, indeed, the EPR-Bohm experiment cannot be explained without them (at least not in a local way).

      Determinism is not an assumption, but as EPR showed, a requirement for keeping the theory local.

      I have yet to see a explanation of this experiment that does not make reference to hidden variables.

      Motl (who has a very similar approach as yours) "explained" EPR by making use of hidden variables (Bertlemann's socks)and then saying that the "quantum" explanation is exacly like that, but, obviously, without the hidden variables. This "quantum" explanation, however, has never been spelled out. How incredibly ridiculous this is!

      Hopefully, you will be the first to succeed.

      Delete
    13. Andrei wrote: "I have yet to see an explanation of this experiment that does not make reference to hidden variables.

      I have different view here: rather than "explaining" the correlations, we should describe them in as simple terms as possible. The idea of photons travelling from the source to the detectors is too much metaphysical baggage that raises many more questions than it answers. I propose to do away with photons, just as we no longer need an aether to "explain" how light propagates.
      (See my post of August 23)

      Delete
    14. Andrea,

      not to let you wait forever: there's nothing wrong with your argument, except that for me it's beside the point. Although we both agree that the mathematical formalism is correct and here to stay, we disagree about what to consider as established facts. You seem to consider Bohr's philosophy as an ultimate "inevitable" truth, established by experimental evidence, whereas I consider a statement like "quantum theory cannot be formulated without the concepts of classical physics" as a proclamation of faith (dogma). Quantum theory needs new, refined concepts that can supplant the classical ones (explain them as emergent).

      Delete
    15. - Werner wrote -
      not to let you wait forever: there's nothing wrong with your argument, except that for me it's beside the point. Although we both agree that the mathematical formalism is correct and here to stay, we disagree about what to consider as established facts. You seem to consider Bohr's philosophy as an ultimate "inevitable" truth, established by experimental evidence, whereas I consider a statement like "quantum theory cannot be formulated without the concepts of classical physics" as a proclamation of faith (dogma). Quantum theory needs new, refined concepts that can supplant the classical ones (explain them as emergent).
      - -
      That's a much better approach. Really appreciated.

      I never claimed that "quantum theory cannot be formulated without the concepts of classical physics", though (although Bohr probably did, possibly meaning something similar to what I am writing below)

      In fact, the concepts of classical physics (e.g. a point in the phase-space of whatever system we are talking about) emerge as a "degeneration" from those of quantum mechanics in the limit when some parameter can be approximated by zero or infinity. Effectively, when hbar in whatever units is sent to zero, with no sweat. What you claim is needed in your last sentence, is there already.

      In practice (I mean the actual practice of using QM as a predictor of correlations), this leads to variances shrinking to zero in the outcome of certain processes - e.g. those that entail observations of position sandwiched between two observations of momentum.

      In QM you only need to assume that you can measure (fully know) position OR momentum; which is also what is strongly suggested by the empirical evidence, not by dogma, by the way.

      Now, the assumption that you can measure (fully know) the position, is not much more mysterious than the *very same* assumption you would need to make were Nature classical and classical mechanics were still a viable option for a fundamental theory of physics.

      In fact, one of the most common fallacies about QM, which I think is what affects our host too, deals precisely with the confusion between the impossibility of knowing *everything* (uncertainty relations) with the impossibility of knowing *anything* (fallacy).

      Questions so far?

      Delete
    16. Werner,

      As Einstein once said, a theory should be as simple as possible but not simpler than that.

      "To explain the observed correlations it seems compelling that something carries information from the source to the detectors: "photons". But we are not permitted to enquire about the properties of those photons, and we must even accept some kind of superluminal conspiracy among them!"

      I disagree with your above conclusion. The assumption that the photon had the observed spin since the moment of emission is a good enough explanation for the observed correlations.

      Andrea, can you address my questions from the August 24 post?

      Thanks!

      Delete
    17. Andrei,

      I do understand the desire to explain the observed correlations, and not everybody will be happy with a decidedly non-local theory. But I also feel that our habits of thought have been steering us away from a concise and unambiguous description of Nature.

      If you accept a non-local theory with signals travelling backwards in time, you can still explain the correlations (in the spirit of the transactional interpretation). I wouldn't call that "simpler than necessary". :-)
      Your preferences may vary.

      Delete
    18. Andrea,

      I share your view of QM as a machinery for calculating correlations. But correlations of what? The ontology of QM is blurred to the extreme. The formulation of the theory could do better than the handwaving about "particles" and "measurement".

      Particle, and with it position and momentum, are classical concepts, and the uncertainty relations are a warning sign that they must be treated with care. For me the gap between the theoretical concept of phase space and the empirical evidence is much larger than for you. There is much room for interpretation in between.

      "One of the most common fallacies ..."

      Reading through the posts, it strikes me that most people seem to think of the Schrödinger equation as describing the deterministic(!) evolution of an individual(!) system. (Yes, this applies to our host, too.) For me, the only way to make sense of the wave function, is to associate it with a statistical ensemble, not a single system. It would be better to say that every physical system is described by a density matrix. Then the statistical character of the theory should be obvious. But then the problem of measurement would suddenly have disappeared. The collapse of the mythical wave function is a red herring.

      Delete
    19. Andrea, Werner, and Andrei,

      Since you three are the last people still commenting on this I'm directing this question at you. There is a post in this thread by Olav Thorsen at 4:23PM August 14 (http://backreaction.blogspot.com/2019/08/the-problem-with-quantum-measurements.html?showComment=1565814236088#c1882875252661697975).

      It appears to be quite straightforward an analysis, and seems to obviate this entire controversy. Essentially he's pointing out that Bell's claim that the classical expectation is linear is incorrect, and that in a correct accounting, the classical prediction is identical to the QM prediction. I'm surprised that none of you find this relevant to your discussion. Did you miss the post or...???

      Delete
    20. Bud rap, I'm surprised you don't seem to know about Bell's inequalities. Olav Thorsen's reasoning rests on wrong assumptions.

      Delete
    21. Werner, since you didn't bother to cite the assumptions to which you object or your reasons for thinking them wrong, I'm left to suspect that you're blowing smoke to cover up the fact that you don't have a defensible position on the matter, just an attitude.

      Prove me wrong. Lay out your objections to Thorsen's reasoning in the form of a rational argument, rather than as an empty assertion.

      Delete
  71. If I know the velocity, then I can predict where a body will get to. Now, can I predict if I cannot see a pattern? So, is velocity a settled pattern of motion? But how did the body fall into that state of motion in the first place? What put the body in that state of motion? Acceleration. And acceleration causes length contraction, time dilation, and mass increase.
    When a human being develops a habit or falls into a habit then we can predict how he will behave based on the habit. Habit means to function in a pattern, and through the habit-forming mechanism the human fell into a pattern of functioning. And we cannot predict if we don’t see or notice a pattern.
    Energy functioning in a pattern is matter. And how did matter get there? Coagulation of energy. Isn’t there an uncanny connection or equivalence between matter, habit and velocity.
    First there is recording, then storing in memory, and then functioning from that memory; this is what causes coagulation of matter; this is what forms habit. Doesn’t this bear an uncanny connection or equivalence with what acceleration does to a body?

    ReplyDelete
  72. Is gravitation a vestige of something? A vestige of something that made the huge mass or body into what it is now? Does gravitation "emerge" as a vestige of the huge mass forming mechanism? The heavier the mass the greater the vestige.

    ReplyDelete
  73. Gravitation and a accelerating rocket have an equivalence according to Eienstien don't they?

    ReplyDelete
  74. Please see my new paper 'Proposal for a new quantum theory of gravity III: Equations for quantum gravity, and the origin of spontaneous localisation' at
    arxiv:1908.04309 Will appreciate your views on it. Thank you.

    ReplyDelete
    Replies
    1. Tejinder Singh,

      Can your briefly summarize how your idea intersects with the measurement problem (per this blogpost)?

      Delete
    2. The theory of spontaneous localisation (also known as GRW theory, collapse models) provides a dynamical, falsifiable solution to the quantum measurement problem. It modifies the Schrodinger equation into a stochastic non-linear equation which implies that the quantum superposition principle is approximate: superpositions are very long-lived for small objects like elementary particles, but extremely short-lived for macroscopic objects such a measuring apparatus. That is why, when a quantum system interacts with a measuring apparatus, superposition is lost, in apparent contradiction with quantum mechanics - this is a dynamical explanation for the so-called `collapse of the wave-function'. The GRW theory is currently being subjected to rigorous experimental tests - it is far from being ruled out yet. However, GRW has been criticised, rightly so, for being ad hoc - it was invented for the express purpose of solving the measurement problem. My new quantum theory of gravity derives GRW as a consequence of the underlying quantum gravity, building on Stephen Adler's work on trace dynamics. Thus the ad hoc nature of GRW is removed. It is important to note that in my theory, spontaneous localisation explains the origin of classical spacetime; resolution of the measurement problem just happens to be a (rather mundane) corollary! For a nontechnical exposition of my work kindly see `Quantum theory and the structure of spacetime' arXiv:1707.01012

      The following is the gist of my quantum gravity theory: "We propose that the universe is made up of `atoms of space-time-matter (STM)'. An STM atom is an elementary particle (fermion) which carries around its own (non-commutative) space-time geometry. The statistical thermodynamics of a large number of STM atoms yields a (falsifiable) quantum theory of gravity. When fluctuations around equilibrium are significant, the theory reduces to classical general relativity, because of a process known as spontaneous localisation. Both gravitation, and quantum theory, are seen as emergent thermodynamic phenomena, whose underlying dynamics is the matrix dynamics of STM atoms. As a corollary, the theory solves the infamous quantum measurement problem."

      Many thanks for your kind interest in my work. Am happy to answer any further questions you might have...Tejinder

      Delete
  75. Quantum mechanics is a deterministic physics. The evolution of the wave function is completely determined. In any quantum basis the amplitudes define probabilities. This is where things get peculiar. For while QM is deterministic it predicts probabilities that in measurement occur randomly. So is QM deterministic or stochastic?

    These interpretations provide some scheme for understanding how a particular eigenstate occurs in a measurement. Decoherence does not do this. All that does is to argue the quantum phase, or the off-diagonal entries in a density matrix, vanish as they are absorbed by a reservoir of ancillary states in an apparatus. That leaves the density matrix as a diagonal matrix of probabilities, but it tells us nothing about how a particular eigenstates occurs. The problem is transformed from a quantum system of amplitudes to classical-like probabilities. Physical systems can be seen as convex sets or hulls, which means the system is L^p or with a measure (x^p + y^p + … z^p)^{1/p}. Quantum mechanics defines a sum over the modulus square of amplitudes sum_nC*_nC_n, and this is effectively an L^2 system because the sum is unity and the square root of unit is again a unit. Classical probability is an L^1 system as just a straight sum of probabilities. So this process is a transformation of a convex set with one measure into another.

    It is interesting to note that for a convex set that is L^p there is a dual set such that L^q such that 1/p + 1/q = 1. For QM this dual set again defines an L^2 measure and it is tempting to think of this as spacetime metric intervals. This comports with my hypothesis that QM and GR are dual systems. Well if we have a map L^2 → L^1 what then is the dual map? Clearly q = ∞, and we have a strange system that has no measure of this sort. It is then tempting to think of this as any sort of deterministic system, whether that be Newtonian-Lagrangian-Hamiltonian mechanics of nonrelativistic particles, or maybe a Turing machine or Conway's Game of Life. These systems are not given by some integration measure, but involve point-like particles, or hard objects like billiard balls, Turing's little cart moving on a tape or in general masses that interact with each other by classical means. Is this connected to GR in some way by this duality? Maybe, for holography shows that physical systems approaching an event horizon become Newtonian in a reduced dimension.

    So back to the issue of quantum measurement. This setting appears to fit with the idea of Zurek's einselection of states, for from a quantum perspective decoherence leaves a set of probabilities, but actual state that occurs is classical-like, at least for a very brief period of time. This process according to Zurek is responsible for defining classical states that are resilient to perturbations. In my setting here it is clear that spacetime physics or gravity plays some dual role, and this is one reason I keep a ear open to the Montevideo Interpretation (MI) and aspects of Penrose's ideas on the reduction of states. This might also be one reason there are spontaneous or stochastic collapses in GRW.

    So with this, I would say that Bohr's interpretation is in some ways not that bad. Indeed with Bohr, Penrose, MI and GRW there is a loss of quantum information, but if we have duality with spacetime physics that lost information may be simply carried away into spacetime as highly IR gravitons. It is not destroyed, but is made very unavailable to any standard quantum measurement. Curiously I am less certain how this works with more ψ-ontic interpretations such as many worlds (MWI). Yet with all of these interpretations one is weaving stochasticity into determinism, mutually contradictory schemes, and with MWI one has the phenomena of stochasticity in this splitting of the world along eigenbases. If we look at this as a map between convex sets we might see interpretations as being neither right or wrong, but as simply model systems that one can choose freely.

    ReplyDelete
  76. It appears that the system has been overwhelmed by the large volume of commentary. Even after "loading more" and refreshing the browser the last post is from 5:57 AM (Tejinder Singh) but email notification shows comments to 9:49. This is a response to Lawrence Crowell's 9:49 post:

    Quantum mechanics is a deterministic physics. The evolution of the wave function is completely determined. In any quantum basis the amplitudes define probabilities. This is where things get peculiar. For while QM is deterministic it predicts probabilities that in measurement occur randomly. So is QM deterministic or stochastic?

    QM is deterministic mathematically. With regard to physics it is indeterminate - that is the central problem. Is the indeterminacy a function of the model or the underlying physics?

    ReplyDelete
    Replies
    1. QM by itself is completely deterministic. QM with measurement introduces stochasticity. The determinism of QM is experimentally supported by an ensemble of measurements, but in any individual measurement the outcome is random. So there is a difference.

      Delete
    2. Sorry, this is semantics. The outcome of any and every individual experiment is indeterminate under QM. Therefore QM cannot be accurately described as completely deterministic.

      Delete
    3. "Therefore QM cannot be accurately described as completely deterministic."

      Unless, of course, the ensemble really exist physically as it does in the MWI.

      Delete
    4. There seems to be a wide-spread misconception about the Schrödinger equation describing the "deterministic" evolution of an *individual* system. (Sabine wrote: "We are talking about one measurement on one particle.")

      This leads to the strange picture of an initially pure state evolving deterministically for a while until it is transformed to a mixture by "decoherence". Finally a measurement occurs that produces a pure state again, and then the cycle repeats.

      It ought to be clear that QM is a statistical theory that always deals with ensembles. One should say that every physical system is described by a density matrix, rather than a wave function. The statistical character of the theory would be made explicit. The closed time-path (Schwinger/Keldysh) formalism shows that unitary evolution and the so-called wave funtion collapse (the Born rule) can be neatly fused together. It does away with the measurement problem, but remains silent about the elements of the ensembles. Quantum ontology is the real problem: what are the "quantum objects" and what are their properties? You can find my answer in my post of August 23.

      Delete
  77. Prof Sabine and antooneo,

    I think you might be slightly mistaken.

    AFAIK, the wavelength of a stationary object is infinite. The frequency is the one that is the one that is reciprocal of mass in that case.

    But even taking this into account, antooneo, it does not affect the actual physical predictions. If you consider the co-moving observer to the electron between the double slit and the screen, then the double slit and the screen are moving, which would have a wavelength good enough to create interference patterns.

    It is not very useful, though, because you have two branches of the electrons passing each slit, and that is just too annoying to consider. The experiment is way easier to consider when the apparatus is considered stationary.

    ReplyDelete
  78. The coming thing in condensed matter physics and quantum engineering is the ability to select the quantum properties of a fermion and/or a boson that are useful and discard the others.

    It is now possible to build a material that hosts quasiparticles that mix and match the quantum properties selected from one or more fundamental particles that are useful and to ignore or restrict the other less advantageous ones. The selected quantum properties can be strengthened and protected while other properties can be ignored.

    Condensed matter Scientists can now see their way in creating Majorana particles because of their potential to store quantum information in a special computation space where quantum information is protected from the environment noise.

    “The new discovery of topological superconductivity in a two-dimensional platform paves the way for building scalable topological qubits to not only store quantum information, but also to manipulate the quantum states that are free of error.”

    ReplyDelete
  79. Andrei,

    We already have experimental evidence of superposition that is macroscopically large. Your argument, if valid, would thus imply a wrong premise.

    Sure, none of them are large and heavy and conscious at the same time, but the burden of proof is really on the people claiming that these things do not obey quantum theory. All we may do is keep raising the upper bound of applicability of quantum theory.

    ReplyDelete
  80. B.F,

    The wavelength of a particle at rest isn't infinite, it's the Compton wave-length and it's the inverse of the mass which is the zero-component of the momentum four-vector. Will you please do me the favor and look it up if you just can't believe it?

    ReplyDelete
  81. Jim,

    Dam sad that Blogspot ate my earlier comment replying to yours.

    Thanks for the clarification, and yes, I very much agree with most of what you say.

    But now that I have been given some time to let the ideas stew, I think it is fair to say that #3 is very much like saying that quantum theory is similar to classical thermodynamics. They are both equally mysterious, silent on most topics, and provide little motivation nor justification. Yet they produce correct predictions in a model-independent manner. And yes, I obviously can agree with this, respect this, and in fact be persuaded that this is a very correct view to take.

    After all, searching my heart, I think I am actually ok with quantum theory being a precursor to something else, being non-real, etc.

    ===

    But it still does not help that we have measurement issues. We still live in a world that have superposition and violate Bell's inequalities.

    However, I am a million times agreeing with you that it is a tremendous shame that people keep thinking that Copenhagen is a monolith. I am always annoyed when people just assume so, especially on surveys. It is about as informative as claiming to be a protestant. Surveys actually need to separate the different types of Copenhagenism before we can have a proper discourse on the relative proportions of people.

    Of course, I only object to certain types of Copenhagenism, but the types I object to are so prevalent...

    ===

    I am not sure why #4 is relevant. Classical thermodynamics is really just a set of experimentally suggested, theoretical self-consistency requirements. They are very insightful and important in practice, doing everything your #4 says, but is not real. As in, #4 is true regardless of a theory's realism status. In fact, it has to be true for any useful theory, and hence is part of the definition of a scientific theory...

    Before I go, may I ask if you are the Jim that is the new and refreshing face of IoP? I really love the huge red ball...

    ReplyDelete
  82. Prof Sabine,

    I am sorry, mea culpa, and thank you for the correction.

    I was thinking of the definition of wavelength in basic QM as h / momentum alone, not considering the photon's measurement of the wavelength.

    Also, sorry for a lot of comments above---I didn't realise that I needed to "load more comments" before seeing them appear.

    ReplyDelete
  83. Sorry for being so late. I kept thinking that my comment didn't get published, when in fact it is that I didn't know I had to load more comments.

    "Quantum mechanics is a deterministic physics."
    "So is QM deterministic or stochastic? "

    This is why I would urge you to please, for goodness, separate what you mean by QM.

    Schroedinger evolution, be it wavefunction or the QFT equivalent, is unitary and deterministic.

    Those interpretations that require collapse, are stochastic. You may say that Copenhagen rules, as used by every interpretation, is stochastic.

    "These interpretations provide some scheme for understanding how a particular eigenstate occurs in a measurement. Decoherence does not do this. All that does is to argue the quantum phase, or the off-diagonal entries in a density matrix, vanish as they are absorbed by a reservoir of ancillary states in an apparatus. That leaves the density matrix as a diagonal matrix of probabilities, but it tells us nothing about how a particular eigenstates occurs. "

    I would point out that the diagonalising of the density matrices is already a satisfactory explanation of why and how a particular eigenstate occurs.

    Which is why I am totally lost when people claim that it is not a solution. I have actually posted my calculations about this below (search c2b), but it is pretty unloved. But it is not important---if you have done the density operators calculation and obtained the diagonalisation, that should already explain how observers see only one eigenstate.

    I admit, however, that the rest of your comments elude me. I am not even sure what you are trying to say. I would, however, ignorantly point out that trying to explain experiments with classical probabilities is wrong. Quantum logic and probability should be used, and I think Reichenbach had solved that part satisfactorily.

    ReplyDelete
  84. Wow, almost 300 comments. So much passion and so little solution;-)

    Please forgive my naivety, I understand practically nothing about this: how the different interactions between the quantum system and the apparatus - or more globally the environment - are qualified? I mean, properties are expressed through interactions, right? Then if you can’t answer how they’re qualified, you can eternally ask what’s happening.

    ReplyDelete
  85. JeanTate,

    Sorry for being so late, I kept not realising that I have to load more comments before being able to see that my replies had been approved and all.

    No, I am saying that current experiments have already been done that show that measurement problems exist in QM if you accept Copenhagen.

    There will always be more new experiments that should be attempted. In fact, I am very curious about what Prof Sabine was talking about, the post-Quantum correlations that QM does not predict yet are hinted at by experiments.

    As for what I meant by experiments have already been done, I mean Bell's theorem et al. I mean, prior to that, people complained that interpretation issues are totally useless. But Bell gave a concrete example of how interpretation issues motivate experimental discovery of interesting experiments to do that tell us way more about quantum theory.

    And in fact, I would even entertain post-quantum hypotheses. After all, the Couder silicone oil drop experiments show how you could have a pilot wave systems underneath QM, that would generate the probabilities that agree with QM qualitatively.

    I simply do not see how you should be coming into a theoretical debate and then trying to shut people up with experiments. Like, you should not respect people who go around maths depts shouting experiments at their desire to know whether Riemann Hypothesis is true until they have found a counterexample.

    ReplyDelete
  86. B.F,

    "We already have experimental evidence of superposition that is macroscopically large."

    Can you provide some examples? I did not say anything about consciousness, it is irrelevant for my argument.

    I did not claim that macroscopic things do not obey quantum theory. They do. Quantum theory says that when a measurement takes place, the system is not in a superposition anymore. The assumption behind Schrodinger's cat experiment is that this prediction of QM can be avoided by placing the system and the instrument inside a box that is supposed to isolate the experiment from the external world. I claim that this assumption is wrong. You can have superpositions only inside the limits of Heisenberg's uncertainty.

    ReplyDelete
  87. isometric,

    I literally explained why Bohmian pilot waves is not classical. Not to mention I also acknowledged that the pilot wave itself is not local.

    ReplyDelete
  88. Sixte,

    If someone accepts that the world is QM and local, then they already do not need to explain Bell's. Bell's assumptions are CLASSICAL, local, realism. At least.

    In fact, Bohm is quantum local realist, and the quantum is precisely why it can get Bell's correct.

    ReplyDelete
  89. Sorry for the late reply. I didn't know how the comment system works here.

    Oh nice, you actually have something interesting.

    I would admit that I am still trying to process what you are saying.

    However, I would not be ok with a polariser as absorbing and re-emitting. Because all systems we have that do a full absorption and then emission, emit them in all other directions at much later times. So, your preferred solution seems to me now to require a vast overhaul of what we typically assume experimental equipment to do.

    Not that that alone is sufficient to refute it, but that such an endeavour would be both expensive and unlikely to yield good results.

    ReplyDelete
  90. Prof Sabine,

    That is extremely sad. Could you briefly describe it, then? Or link to the places in your blog that does? Or write a new blog post about this? Literally grasping at straws here.

    Peter Shor (are you the same as the legendary Shor's algorithm?),

    I literally do not care what kind of extra stuff is needed to be post-quantum. Who doesn't want to do an experiment showing either post-quantum (and hence Nobel) or upholds quantum (hence worthwhile to do anyway)? The experiments are always king!

    ReplyDelete
  91. Always liked reading the blogs, thank you.
    One may interpret QM as Brownian, in a fictitious medium, which has unique properties - strictly Markovian BM requires the same infinite propagation velocities for disturbances to equilibrate the bath, similar to wave function collapse.

    ReplyDelete
  92. The difference is you have interposed 'human actors' who make the decision to set up the probability problem. Which 'actors' are making electrons behave probabilistically? Those are the 'hidden variables' but which must communicate instantaneously or noncausally giving us the measurement or collapse problems.

    ReplyDelete
  93. About the wave-length of a particle (I do not know how to make the correct position reference here):

    The wave-length in the Schrödinger equation and in the Dirac equation is not the Compton wave-length but the de Broglie wave-length. I have again consulted several textbooks for this and found it confirmed.
    And if you look into de Brogie's famous deduction of the wave-length, you will find it in the same way.

    To make it clear: The use of lambda = h/p is physically incorrect by my understanding. But it is present main stream. I have also given several talks about this at physical conferences and I never heart your argument.

    ReplyDelete
    Replies
    1. antooneo,

      The particle interacts with the slitted barrier, not with the observer. So, in order to have interference the particle should display a suitable wavelength in the reference frame of the barrier, not of the observer.
      If the particle does not move relative to the barrier then you will not have interference, as expected. What is the paradox?

      Delete
    2. You just stumbled over the apparent clash between de Broglie and Galileo – read this here (enlarge for more).

      Delete
  94. On wavelengths. Compton wavelength is related to the local mass energy possible to be released from exited bounded states of matter for a photon but de Broglie wavelength depends on the difference between the frame of measurement device and the frame of an object for seeing some interference pattern. They are not commensurate to compare.

    ReplyDelete
  95. Answering Jean Tate, August 17, 2019: SQPR relates to the traditional measurement problem in Quantum Mechanics by complexifying it: Devoret and Al. discovered that measuring Quantum Jumps is not necessarily a fundamental measurement, because some Quantum Jumps are preceded by a signal which can be picked up (and reversed!)

    Thus Devoret demonstrated that there are hidden variables, at least in the case of some Quantum Jumps. So, at the very least, there are measurements which are more fundamental than others: this addresses directly the Schrodinger cat hierarchy problem: that is, what is the most fundamental wave function... Is there such a thing? Devoret showed that all and any Quantum Jump is not fundamental: some may have inner machinery hidden inside. Next question: do they all?

    In any case, this is real physics, pondering what reality is, trying to answer real questions, such as what is really going on, when one measures. Bigger accelerators to find out about non-existing problems, as Sabine points out, is not how to seriate inquiry, indeed!

    ReplyDelete
  96. Andrei:

    The paradox comes from the "relativity principle". In the frame of the barrier there is no paradox, true! But by relativity, also in the frame of the co-moving observer everything should develop according to known physics. In his frame, however, the de Broglie wavelength is infinite and so all points of the wave are having the same phase everywhere in space. Therefore a destructive interference is not possible at any place and a pattern cannot occur for this observer.

    But we know that the pattern occurs also in the view of this observer; that is the paradox.

    ReplyDelete
    Replies
    1. As far as I know you cannot get interference with beam of particles in a momentum eigenstate. Before the barrier you let the beam pass through a narrow slit that plays the role of a position measurement device, so that your particles have an uncertain momentum in the plane of the slits. Even if you start with particles with known momentum you cannot know the momentum component in the direction of the beam. So, there is no way you can find a frame of reference where the particles in this experiment are stationary.

      Delete
  97. Andrei,

    Basically any experiment using anything large is sufficient. So, things like SQUIDS, cavities, and if you just want size but not mass, you can have the standard double slit. The interfering wavefunction is huge and totally bigger than my finger.

    As for the assumptions behind Schroedinger's Cat, the analogy is just a bad analogy. We already observe plenty of stuff that are necessarily superpositions, not least because physics is basis-independent. You needed to at least say that measurements in QM stop being superposed amongst the different measurement outcomes. That, however, still does not require collapse, because decoherence is sufficient within unitary (Schroedinger) evolution to explain.

    Case in point:
    "You can have superpositions only inside the limits of Heisenberg's uncertainty."
    By that you mean you don't consider entanglement a refutation of this precise statement? Or rather, what do you even mean by Heisenberg's uncertainty here? There is no measurement that is in violation of it.

    ReplyDelete
  98. Sabine wrote: "either way you turn the pieces, they won't fit together"

    Perhaps we've been looking at the wrong pieces for too long?
    Strangely, many people believe that "every physical system is described by a wavefunction". Schrödinger's equation claims continuous and deterministic evolution of "something", whereas discontinuity and randomness are the hallmarks of quantum physics. Shouldn't that give you pause? And isn't Schrödinger's cat reductio ad absurdum of this idea?

    Of course, the orthodoxy denies any paradoxes.
    "QM is absurd. But that's okay - it has to be."
    Under the spell of Bohr the theory has acquired transcendence.
    "QM cannot be formulated without the concepts of classical physics." Slightly paraphrased: the human mind cannot possibly find appropriate concepts that would allow QM to be formulated in natural terms. What is this claim based upon?

    It is embarrassing that after nine decades the problem is still unsolved. (And so much heat be generated in this discusion group.) I share your view that there is a problem, and I am optimistic that it can soon be solved. The present situation in quantum theory is not unlike that of electrodynamics before 1905: flawless mathematics, and passing all experimental with flying colours. But the physicists of the time find the theory extremely "unanschaulich", riddled with paradoxes, and the mathematics arcane. Since it is highly successful, the solution of the problem should not require a new theory, but rather a refinement of its concepts (what is it about?) and the removal of awkward metaphysical baggage (like the "ether" or "photons").

    My optimism is based on an idea I tried to publish 32 years ago. I have it on paper that it is not worth publishing - but perhaps the audience here is more receptive. :-)
    I'd be happy to provide more details.

    ReplyDelete
  99. Regarding your objection to MWI you say "local observers are spontaneously carried along with one state or path." That seems to be simply a restatement of the MWI, I don't understand how that constitutes a flaw.

    ReplyDelete
  100. Sabine wrote:

    "please write down a definition for "detector" in many worlds."

    A detector is just an Hermitian operator. Each eigenstate corresponds to a "world" with a corresponding eigenvalue. Whether the interference pattern would be maintained depends on how much information there is in the measurement.

    For example, in the double slit experiment you can measure the position X, which can tell you which slit the particle went through. If the potential in Schrodinger's equation is not a function of X, then there is no actual measurement taking place and you would maintain the interference pattern. Still, in the MWI you can say that in one world X measured x_1 and in the other other world x_2 was measured.

    Then you can add a potential term that depends on X. For example, by adding a test mass in the middle and adding the relevant gravitational potential. The position of the test mass would have some quantum distribution.

    Now you can turn the measurement on and off simply by filtering your experiment according to the position of the test mass. If you choose only the states where
    the test mass is exactly in the center, there is no information in detector and the interference pattern would be maintained.

    If you choose only states where the test mass is at least 5 sigma to the right, the pattern on the screen would correspond to the case where all particles went through the right slit.

    Finally, you can replace the test mass with a macroscopic detector and get exactly the same behavior.

    The only thing that MWI says is that there is no wave function collapse. The detector's reading is always "quantum".

    ReplyDelete
  101. Udi,

    "A detector is just an Hermitian operator. Each eigenstate corresponds to a "world" with a corresponding eigenvalue."

    This does not follow from the Schroedinger equation and it's logically equivalent to the measurement postulate.

    ReplyDelete
  102. In the case of the goat behind the curtain, we have a clear and intuitive sense of how we transitioned from the possible to the actual.

    ReplyDelete
  103. Psi (Ψ) - in the light of quantum behavior.
    The Problem with Quantum Measurements - Psi (Ψ)
    The '' measurement problem'' / ''wave-particle collapse''
    #
    There isn't electric wave without quantum particle.
    The wave-function is result of a real work of quantum particle (h)
    The wave-function Psi (Ψ) is a derivative form of quantum particle.
    The wave-particle collapse problem could be contemplated as
    boundary changes of wave and particle simultaneously.
    #
    When the wave collapses, the pure electric particle (E=h*f)
    changes its parameters into negative potential state - Dirac's
    virtual / antiparticles (-E=Mc^2) and "disappears " into Zero Vacuum T=0K.
    =======

    ReplyDelete
  104. Sabine wrote:

    "This does not follow from the Schroedinger equation and it's logically equivalent to the measurement postulate."

    I am not a big fan of MWI for the same reason that I'm not a fan of any interpretation -- because there is no empirical evidence in the interpretation.

    Saying that. In MWI there is no measurement postulate. There is only the wave function |Psi>. If you choose a basis, say the position X, You will get <X|Psi>=Psi(x) that describes the particle at different positions. If you choose the momentum basis, you will get <P|Psi>=Psi(p) that describes the particle at different
    momenta.

    Until now there was no measurement and there is no preferred basis. Now, if I add a measurement device that measures X (like the test mass in my previous post), then that device, by construction responds to the position of the particle and therefore that is what it measures.

    In this description there is only the time evolution of the wave function by the Hamiltonian, so clearly, there is no wave function collapse.

    ReplyDelete
  105. Udi,

    "Now, if I add a measurement device that measures X..."

    You just referred to the measurement device, which requires you to draw on a definition for that which, as I explained above already, is logically equivalent to the measurement postulate.

    Whether you interpret it the same way is entirely irrelevant for the calculation. My point is that postulating "what I call detector at t_e is only one branch of what I formerly called detector at t_i<t_e" has the exact same result as postulating that "at t_e I update the probability".

    The relevant point is that MWI is not, as most people tend to think, axiomatically any simpler than Copenhagen. It merely shoves the measurement postulate into a postulate about what constitutes a detector.

    ReplyDelete
  106. It shouldn't be necessary in this discussion group to elaborate on Schrödinger's cat. But judging from some of the comments, maybe yes.

    Have you noticed that discussions of the paradox invariably involve the most notorious of all irrational numbers, the square root of two? Of course you have, and you know it's because of the holy principle of unitarity. But you never see a minus sign, or even an imaginary i between the two kets. According to the rules, these would be different states. Of course you could absorb the phase factors in the definition of the kets /dead) and /alive). But are these really *two* states?

    It is distracting that Schrödinger constructed such a ridiculous case. He might as well have chosen a calorimeter, where as a result of the decay a drop of water was heated from 14 to 15 degrees Celsius. Would people constructing kets from dead and alive cats dare to write down
    ( /14 C) + /15 C) ) / sqrt(2) ?
    This clearly is an abuse of notation. For a drop of 1 cubic millimetre you find from Boltzmann's S = k*ln W an entropy increase S/k of about 10^18. And that is just the *logarithm* of the number of states! The square root of 2 is clearly suspicious.

    It ought to be clear that real measurements always involve irreversibility (and are certainly not instantaneous).
    Is photon absorption an irreversible process?
    Does it always imply the collapse of a wave function?

    Such questions are more reasonably discussed in the context of quantum field theory and quantum statistical mechanics. Many physicists probably think that if QM is beyond human understanding, then QFT must be even more so. But it should be closer to the heart of the problem.

    ReplyDelete
    Replies
    1. Is photon absorption an irreversible process?
      Does it always imply the collapse of a wave function?
      / Werner7:41 AM, August 20, 2019 /

      Does photon's absorption/emission have connection with collapse of its wave ?
      ===

      Delete
    2. No, it is not an irreversible process. There are no fundamentally irreversible processes.

      Delete
  107. B.F.

    The interference in the double-slit experiment is based on the deBrogle wavelenth of that particle which is related to the uncertainty principle. The more massive a particle is the smaller is the wavelength so the smaller the separation between the slits should be to see interference.

    QM guarantees us that as long as the superposition is prepared by a measurement of a non-commuting observable such a state would not lead to any observational disagreement. If a particle's position is measured by passing it through a narrow slit strongly fixed by the table there is no way to accurately measure particle's momentum (without changing the experimental setup). But consider the case of a cat that moves inside a box. I place a dust grain in orbit around that box and I look at its orbit. If the cat is sitting at the center of the box (and the box is a perfect sphere) a perfectly circular orbit of the sand grain is possible. If the cat moves in another place the mass distribution inside the box changes and this will perturb the orbit of the grain. I am still limited in my ability to measure the grain by the uncertainty principle, so I cannot measure the position of the cat with a better accuracy than that. But there is no way I could not distinguish between two positions of the cat separated by say, 1m.

    If the above procedure works it means that you cannot prepare a cat superposition that goes beyond the uncertainty limit which for a cat-sized object is practically 0. If you claim that my proposed schema does not work, please point out the error.

    Also, there is nothing special about a sand grain, in principle it can be any object with mass. So, my argument goes, macroscopic superpositions are not compatible with any massive objects around them, including the outside observer. So, they just cannot exist.

    ReplyDelete
  108. Sabine, JeanTate, thanks for these interesting comments and answers.
    Which (yet) thought experiments are you referring to, Sabine? Would you mind sharing a reference? Could you point out a good paper about NISQ, JeanTate?

    ReplyDelete
  109. Dr. Hossenfelder: There are no fundamentally irreversible processes.

    Isn't wave function collapse an irreversible process? Regardless of what causes wave function collapse (or if it is an uncaused event)?

    A particular value of many potential values was selected, and there is no way to return the particle to its original state of multiple superpositions.

    That's a real question, I don't know the answer.

    ReplyDelete
    Replies
    1. Dr AM Castaldo,

      Yes, of course, sorry. I meant, there are no fundamentally irreversible processes except for the measurement process in quantum mechanics. (Though the irreversibility isn't the problem.)

      Delete
    2. Sabine,

      „Though the irreversibility isn't the problem.“
      With problem you probably mean the problematic relation of the measurement to: 1.) linearity in the deterministic, reversible, unitary evolution. 2.) locality of SR or better jumping and superposed mass with respect to GR, 3.) reductionism.

      “This strongly suggests that the measurement is an effective description of some underlying non-linear process, something we haven’t yet figured out.”
      What could be more non-linear as a simple threshold? A threshold that would be forced on us by the assumption that gravity cannot be quantized.

      Delete
    3. "no fundamentally irreversible processes except for the measurement process in quantum mechanics"

      I share the view that irreversibility is a feature of the macroworld. But where is the boundary between the micro- and the macroworld? In the everyday world absorbed light turns into heat. But in the Aspect et al. experiment the absorption of a photon is what? Reversible? Or a special fundamentally irreversible process?

      The trouble is that, according to Bohrian dogma, "measurement" is a fundamental notion. There is no definition of what a measurement is, except that it always yields a classical result. As John Bell has reiterated again and again: the basis of quantum theory should be formulated without the term "measurement".

      This applies to the term "knowledge", too. It appears 90 times in this discussion.

      Sabine: Is "measurement" a fundamental process for you?

      Delete
  110. Photon can be absorbed and photon can be emitted
    Photon cannot be absorbed for ever . . .
    . . . sooner or later it will be emitted
    After process ''absorption '' goes process ''emission'' and vice versa
    In Nature this process is reversible
    Nature cannot exist without Light / Photon
    Everything in Nature has its period of life --
    -- period between ''absorption'' light and its ''emission''
    The process between ''absorption - emission'' is math linear.
    The process that underlying between ''emission - absorption''
    is hidden in non-linear mathematics.
    =====

    ReplyDelete
    Replies
    1. An absorbed photon has active X parameters,
      an emitted photon has potential Y parameters
      The unity between X and Y is disclosed by
      '‘The Law of mass/energy conservation and transformation’'
      ====

      Delete
  111. Sabine wrote:

    "You just referred to the measurement device, which requires you to draw on a definition for that which, as I explained above already, is logically equivalent to the measurement postulate."

    I was very explicit in my previous post, so I will repeat myself.

    I'm looking at the double slit experiment. My measurement device is a test mass in the center, between the two slits. I add a gravitational interaction term between the double slit particle and the test mass. All I have is a Hamiltonian

    "Whether you interpret it the same way is entirely irrelevant for the calculation. My point is that postulating "what I call detector at t_e is only one branch of what I formerly called detector at t_i<t_e" has the exact same result as postulating that "at t_e I update the probability"."

    I have no t_e in my Hamiltonian. The potential term is static.

    I choose initial conditions of an incoming particle with fixed momenta. For simplicity, I keep the potential constant behind the double slit, so a free particle is still a solution there.

    After calculating the wave function psi(x,xm), where xm is the coordinate of the test mass, I can "turn the measurement on and off" by deciding what part of the wave function I am looking at. If I look at psi(x,xm=0), I would get an interference pattern. If I look at psi(x, xm>5 sigma), I would see a pattern of the particle going only through the right slit.

    So I have a "quantum" measurement device whose position tells me which slit the particle passed trough.

    ReplyDelete
  112. Udi,

    Thanks for your patience, but I do not understand what the point is of your comment. I am telling you that Many Worlds does not solve the measurement problem but instead reformulates it by way of postulating what we mean by "detector". I don't know what you are talking about.

    ReplyDelete
  113. Sabine,

    You are telling me that Many Worlds does not solve the measurement problem, but you are not explaining where the problem is.

    I understand that claiming that the wave function collapses when you make a measurement is problematic -- it is not part of the quantum formulation, it is not local, it is irreversible and it is random.

    In Many Worlds there is not wave function collapse and all these problems go away. Now the theory is self-contained, local, time-reversible and deterministic.

    ReplyDelete
  114. Udi,

    I have explained in my blogpost what the problem is. If you think the wave-function is real, then you need to update it upon measurement and the measurement process is non-linear, hence incompatible with the Schroedinger equation. If you don't think it's real, you have a problem with reductionism.

    If you do not understand this argument, then please tell me what it is that you do not understand. Simply claiming I didn't explain it doesn't work, because arguably I did. That's why you are here.

    As to many worlds, I already said this above. In many worlds you have to postulate what a detector is, which is logically equivalent to the measurement postulate. It is beyond me why anyone thinks this solves a problem because you see when you make a calculation that it is, of course, exactly the same thing. That's why it's called an interpretation.

    ReplyDelete
  115. Udi Fuchs,

    As far as I know there is no way to get the Born rule in MWI. So, it cannot be considered equivalent to QM. It is just a hypothesis that, as of now, did not worked out.

    ReplyDelete
  116. Consider the Aspect et al. experiment. We can probably agree on the following realist description: electrons in a calcium atom do some "wiggling", and a few meters away, and a few nanoseconds later, electrons in the detectors do some similar wiggling.

    To explain the observed correlations it seems compelling that something carries information from the source to the detectors: "photons". But we are not permitted to enquire about the properties of those photons, and we must even accept some kind of superluminal conspiracy among them!

    The easiest way to avoid this conundrum is to discard the idea that something is traveling from the source to the detectors. Only the emission and absorption events are real. And the correlations between events separated in space and time is a fact of nature that we should just accept and describe in as simple terms as possible, if "explaining" them throws up many more questions than it answers.

    In electrodynamics before 1905 physicists faced a similar problem: that light could propagate without an ether was inconceivable, yet the implied "ether drift" failed to show up. Today we interpret the Michelson-Morley experiment as evidence against the existence of the ether. Should we not view the Aspect et al. experiments as evidence against the existence of photons? At least of photons with the connotation of "quantum object moving through space-time". Only the notion of photons as pairs of absorption/emision events should survive.

    Of course, electrons need to be considered, too. At the zeptosecond scale the world line of an electron would no longer look continuous, but become a "dotted line", with the spacing of the dots defined by the 511 keV scale. (Not some ad hoc "collapse parameter" as in GRW.) Photon and electron fields can be changed by arbitrary gauge transformations -- this could suggest that only their mutual interaction events are real.

    So the basic idea is:
    * Matter is made of atoms.
    * Atoms are discrete in space, but continuous in time.
    ! Extend the idea of atomism from space to space-time.

    QM has never been precise as to what it is about: neither waves nor particles, but a combination of both. The beauty of the "event picture" is that it provides a true synthesis of the wave/particle concepts. Particles and waves are special (emergent) patterns of events in space-time. Quantum field theory is a statistical theory of events and the correlations between them. Fields describe the correlations of events. From the action functional all correlation functions can be derived.

    At this stage the theory is just an empty shell. I should say a lot more about the structure of an "emission" event, which certainly comprises more than a single point in space-time. Lest the lines limit hit me, I defer this to a separate posting.

    1/2

    ReplyDelete
    Replies
    1. A photon is not created in an instant. An emission process must correspond to a definite pattern of events. What that pattern might look like can be gleaned from the Kubo-formulas for absorption and scattering coefficients, which express them as Fourier integrals of the current density fluctuations. These can be evaluated using the closed time-path (Schwinger/Keldysh) formalism. You can find a rather pedestrian application of that formalism in my paper J.Phys. A 21, 407-418 (1988). Looking at eqs. 5+6 I find it suggestive to think of an emission event as two successive event pairs (currents, representing the motion of an electron) on alternate branches of the time-path.

      It is significant that the closed time-path formalism directly yields probabilities, rather than probability amplitudes. The square of the usual amplitude (Born's rule) arises automatically, because propagators on the forward time branch are accompanied by corresponding ones on the backward branch. They are the "confirmation waves" of the transactional interpretation, and they ensure "consistent histories". But one should not think of them as real waves; they are merely part of a mathematical apparatus describing correlations between events.

      I haven't been able to construct a stochastic process exhibiting the properties of a Dirac electron, but I'm pretty sure it can be found. The problem with spontaneous localization (GRW) is that it ruins momentum conservation and leaves us with a process resembling diffusion. But event pairs on the forward/backward time branches could provide the necessary rigidity, so that the resulting world lines would look straight (in the absence of fields).

      Apart from a tag defining which time branch an event exists on, events should also carry a phase factor, i.e. we need to study point processes in a 5-dimensional manifold, if we want to arrive at a stochastic model with the features of QED. The vector potential A could then be interpreted as the average shift of phase as one passes from an event on the forward time branch to its twin on the backward branch. The phase itself, although defined only up to a gauge transformation, can become a physical observable, as happens in a superconductor.

      Of course, these are just speculations. But the huge gap in my argument also offers a fresh perspective in the stale debate about the interpretation of QM, and deserves to be looked into. It would be sad indeed if in 2025 we should still be debating wave function collapse and the quantum measurement problem.

      2/2

      Delete
  117. Take a cat in the Schrodinger wave Psi (Ψ)
    The cat exist in the sphere of probability density = |Psi|^2
    Find cat when the Schrodinger wave collapses
    Then the cat exist in the math method of ''Renormalization''
    #
    In Maxwell EM waves there is Lorentz electron force
    What force exists in the Schrodinger wave Psi (Ψ) ?
    ========

    ReplyDelete
  118. We should get over discussing interpretations of quantum mechanics. We have been stuck there for a century now. The interaction of a quantum system with the so-called measuring apparatus is a dynamical process which should be explained by precise, falsifiable, mathematical equations. The theory of spontaneous collapse does just that. Recently we have shown that spontaneous collapse is a consequence of a new quantum theory of gravity. The same process which explains how classical spacetime and matter emerges from quantum gravity, also solves the measurement problem. There is no longer any need for one or the other interpretation of quantum mechanics. The Schrodinger equation is shown to be an approximation to a deeper theory. See arxiv:1908.04309

    ReplyDelete
  119. - Sabine wrote in reply to Udi -
    If you don't think it's real [the wavefunction], you have a problem with reductionism.

    If you do not understand this argument, then please tell me what it is that you do not understand.
    --

    Yes, please do expand on this, as I don't understand it at all. This point is also mentioned in the main post and quite glossed over in there too.

    The only clash I can think of between some form of reductionism and quantum mechanics specifically - i.e. entangled systems not admitting maximally complete descriptions as independent entities despite their physical separation - is unequivocally problem-free, in the sense that this kind of reductionism has been empirically disproven for at least a few decades now. No big deal, if you ask me, because physics is still local.

    Beyond that, I am in the total dark about what you might mean with that "problem with reductionism", however "reductionism" is defined, that would not apply to classical mechanics too.

    Thanks

    ReplyDelete
  120. Sabine wrote:

    “If you think the wave-function is real, then you need to update it upon measurement and the measurement process is non-linear, hence incompatible with the Schroedinger equation.”

    In MWI the wave-function is real. Measurement does not change the wave-function, so everything is compatible with Schroedinger’s equation.

    “As to many worlds, I already said this above. In many worlds you have to postulate what a detector is, which is logically equivalent to the measurement postulate.”

    The detector is just another term in the Hamiltonian, so there is no need for any measurement postulate.

    Andrei wrote:

    “As far as I know there is no way to get the Born rule in MWI. So, it cannot be considered equivalent to QM.”

    The Born rule is one of the postulates of quantum mechanics. I am not aware of any interpretation that derives it.

    ReplyDelete
  121. Andrea,

    If you have a reductionist theory, then any macroscopic objects or processes should be derivable from the microscopic ones. For this reason the axioms of your theory should not contain reference to anything like "measurement" or "knowledge" or "detector".

    ReplyDelete
  122. Udi,

    "The detector is just another term in the Hamiltonian"

    It is not. A detector is made of particles. It has a wave-function. Try to figure out what the wave-function is, then maybe you will see what I am telling you.

    ReplyDelete
  123. Udi Fuchs,

    "The Born rule is one of the postulates of quantum mechanics. I am not aware of any interpretation that derives it."

    Yes, but the other interpretations do not claim that the wavefunction is all that exist. If Born postulate cannot be derived out of the wavefunction it implies that this basic assumption of MWI is wrong.

    ReplyDelete
  124. - Sabine wrote -
    If you have a reductionist theory, then any macroscopic objects or processes should be derivable from the microscopic ones.
    --

    Hi Sabine,

    Still unclear to me, sorry.

    If I interpret the macroscopic/microscopic distinction in the usual sense of thermodynamics, then the sentence is evidently as true for QM as it is for CM, is it not?

    If I interpret it in the sense of elementary/composite systems, then reductionism applies to Nature as much as fasterthanlightism does, as per the previous post.

    ... or maybe you just agree that QM is reductionistic in the first sense and the crucial point was in the unquoted sentence?

    Can you please expand?

    Thanks

    ReplyDelete
  125. Andrea,

    No, in QM you cannot derive what the detector does from the microscopic laws. You need an extra assumption that is *incompatible* with the microscopic law. As I wrote in my blogpost.

    ReplyDelete
  126. - Sabine wrote -
    No, in QM you cannot derive what the detector does from the microscopic laws.
    --

    What?! You need to be a lot more specific on this.

    A Wilson chamber, say, can be described microscopically as a region of space filled with a gas of some composition in some metastable state (you can calculate the statistical matrix of the gas as function of the macroscopic properties, in principle), that is "poked" periodically with a stream of photons in a roughly known translational state (again, you can calculate the statistical matrix of the electromagnetic field in the region as function of the macroscopic properties of the emitter).

    The end result of each "poking" event (i.e. taking a picture) is given by one individual sample of some photosensitive material of some composition, that was known to be in some other configuration with similar microscopic descriptions in terms of coarser thermodynamic properties.

    It honestly looks to me I just did what you said can't be done. What am I missing?

    - Sabine also wrote -
    You need an extra assumption that is *incompatible* with the microscopic law. As I wrote in my blogpost.
    --

    If you are referring to the fact that the Schroedinger equation only has a time limited validity, I also explained in my very first reply why this complaint is not even wrong. I'll be happy to make another summary, if this is the case.

    ReplyDelete
  127. Andrea,

    The Schroedinger equation does not give you a measurement outcome. I explained this in my blogpost.

    ReplyDelete
  128. - Sabine wrote -
    The Schroedinger equation does not give you a measurement outcome. I explained this in my blogpost.
    --

    Correct, the Schroedinger equation *takes from* you a measurement outcome. It is Nature that *gives* outcomes back. As per QM.

    Can't see the relation with what we were discussing

    ReplyDelete
  129. May I try to alleviate the confusion?

    It is a common misconception to think of the wave function as describing an *individual* quantum system. People tend to forget that QM is a statistical theory. A ket by itself is meaningless - it always has to be combined with a bra and integrated over.

    People working in the Heisenberg picture never have this problem, because the state vector is constant. And they never associate a matrix with an individual system, but always with an ensemble.

    The "deterministic" evolution applies only to the individual members of the ensemble. It has to be combined with the Born rule to arrive at the full picture. The closed time-path (Schwinger/Keldysh) formalism in quantum statistical mechanics seamlessly combines the "deterministic" evolution with the Born rule.

    Or does this, perhaps, add to the confusion?

    ReplyDelete
  130. Sabine wrote:

    “It is not. A detector is made of particles. It has a wave-function. Try to figure out what the wave-function is, then maybe you will see what I am telling you.”

    That is exactly what I did in previous posts. I described a detector made out of a single test mass to measure which slit the particle of the double slit experiment passed through.

    “If you have a reductionist theory, then any macroscopic objects or processes should be derivable from the microscopic ones.”

    I think that our misunderstanding stems from the fact that you think that a detector should be macroscopic while I insist on looking at microscopic detectors. I understand that some people think that wave function collapse should be a macroscopic event, but in MWI there is no wave function collapse.

    If you insist, I could implement the X operator for measuring the position of an electron using a phosphorus screen. I would model each pixel with a test mass similar to the one I suggested above and write an explicit Hamiltonian for it. It would behave exactly like we expect a phosphorus screen should behave.

    Andrei wrote:

    “Yes, but the other interpretations do not claim that the wavefunction is all that exist. If Born postulate cannot be derived out of the wavefunction it implies that this basic assumption of MWI is wrong.”

    In MWI the wave function is all that exist. That is why we say that in some worlds the spin is up and in others it is down. You can still ask what is the ratio of worlds in which the spin is up and the answer is the square of the absolute of the wave function. Essentially, it is the Born postulate with the word “probability” replaced with “ratio”.

    ReplyDelete
  131. Udi quoted Sabine: "A detector is made of atoms. It has a wave-function."

    As I tried to explain in the preceding post: the wave-function is red herring. The discussion should focus on the ontology of QM, not the mythical wave-function and its collapse. Or the boundary between microscopic and macroscopic. Please have a look at my post of August 23.

    ReplyDelete
  132. Udi Fuchs,

    "You can still ask what is the ratio of worlds in which the spin is up and the answer is the square of the absolute of the wave function. Essentially, it is the Born postulate with the word “probability” replaced with “ratio”."

    You cannot postulate features that can be derived. It is like postulating the area of a sphere in Euclidean geometry, or like string theorist postulating the existence of a vacuum that corresponds to the standard model.

    Why should a hypothetical inhabitant of a MWI world percieve such a ratio of worlds?

    MWI has also the problem of explaining the reason we percieve the world as we do (objects moving in space). Other interpretations do not need such an explanation because this basic feature of the world is part of their ontology.

    ReplyDelete
  133. B.F.

    My understanding is that Sabine does not support the MWI. In fact here is a quote from her from another comment:

    "The many worlds interpretation replaces the measurement postulate by postulate about what constitutes a detector. This doesn't solve any problem, it merely renames it. If you do not understand what I say, please write down a definition for "detector" in many worlds."

    So my question is about whether she believes that some "extension" is possible, rather than just another new Interpretation (and certainly not MWI). If so what is the nature of that extension?

    I have just read her blog about SuperDeterminism: maybe my answer is in there somewhere?

    ReplyDelete
  134. Andrei wrote:

    “If Born postulate cannot be derived out of the wavefunction it implies that this basic assumption of MWI is wrong.”

    Andrei also wrote:

    “You cannot postulate features that can be derived.”

    Make up your mind. The born rule can or cannot be derived? Actually, I don’t really care what are the postulates and what is derived. Quantum mechanics is consistent. If you claim otherwise, show me the inconsistency.

    “Why should a hypothetical inhabitant of a MWI world percieve such a ratio of worlds?”

    Luckily, we live in such a world, so I can tell you from experience. We perceive those ratios as probabilities.

    ReplyDelete
  135. Udi,

    "I think that our misunderstanding stems from the fact that you think that a detector should be macroscopic while I insist on looking at microscopic detectors."

    If you define "detector" as something that does not make a measurement, then you have no problem with the measurement that such a detector makes. That's correct.

    ReplyDelete
  136. Udi Fuchs,

    MWI makes the following implicit assumptions:

    1. The wavefunction (being the only existing entity) is capable "producing" a world like ours, where observers exist and can do experiments.

    2. Assuming 1 is true it is also true that those observers will register experimental results in agreement with Born's rule.

    Now, if MWI is true, it should be possible to deduce both 1 and 2 because there is simply no other fundamental structure available, right? This is logically equivalent to saying that if either 1 or 2 cannot be deduced from the fundamental entity (the wavefunction) MWI is false. Postulating 1 or 2 is equivalent to postulating that MWI is true, which is absurd.

    Let me repeat this argument for string theory.

    If string theory is true there must be a vacuum that corresponds to our world. This is logically equivalent to saying that if no such such vacuum exists string theory is false. So, a string theorist cannot postulate the existence of this vacuum because that would be equivalent to postulating that string theory is true.

    And also:

    If classical electromagnetism is true, the hydrogen atom, as described by the theory's equations should be stable and it should have a discrete energy spectrum. This is logically equivalent to saying that if a stable hydrogen atom cannot be deduced from the theory, classical electromagnetism is false. One cannot postulate that the classical atom is stable, that would be absurd.

    So, you can only postulate properties/entities that are independent from the other postulates. If they can be derived, they should be derived. Proceeding otherwise can lead to inconsistencies or absurdities. In Bohm's theory for example the Born rule is obtained from the particle distribution. This is consistent because this distribution is independent from the wave function.

    ReplyDelete
  137. "Bohm QM runs into trouble with relativity."

    No. All you need is to accept a preferred frame (that means, accept the Lorentz ether interpretation of relativity), and to use field ontology for relativistic field theory. That means, the configuration space should be the field, not particles.

    BM interprets the wave function as ontic, but there is no necessity for this. In a variant of Stochastic Mechanics, Caticha's Entropic Dynamics, the wave function is completely epistemic.

    "None of these has a solution to the measurement problem."

    Wrong. All the realistic interpretations (Bohm, Nelson, Caticha) which have, beyond the wave function also a configuration space trajectory q(t), solve the measurement problem in a straightforward way: What we observe is the q(t) of the measurement device. The wave function of the quantum system is the effective wave function, obtained by putting q(t) of the measurement device into the wave function of the full configuration (quantum system + measurement device).

    ReplyDelete
  138. A preferred frame in relativity is a big problem that I will not go to. Interpretations in a sense solve the measurement problem by proposing something that is itself a big problem. That is called begging the question.

    ReplyDelete
  139. - Sabine wrote -
    If you want to deny the conclusion you have to find a mistake in my argument
    --
    I’ve been doing exactly so for almost a week now.

    For the third time: the “mistake” in your argument is when you claim that knowledge being emergent is problematic for the psi-epistemic interpretation (QM for short) because it leads to some breakdown of reductionism.

    I wrote “mistake” in quotes because I’m still not clear (and asked twice already!) what you mean exactly with breakdown of reductionism; in any case, I offered two understandings: one that is manifestly problem-free and one that makes QM just as problem-free as classical mechanics would be.

    This is *my* argument; details in the very first reply, and if some implications not spelled out are not self-evident, I am here: the magic words are “please expand”, accompanied if possible by some speculations about the meaning of what is not clear.

    Otherwise, please follow your own advice quoted at the top...

    ReplyDelete
  140. Andrea,

    I already said this several times, here we go once again: A reductionist theory is one in which macroscopic concepts can be derived from the microscopic ones. If you have a postulate (measurement postulate) that does not only not derive from the microscopic law, but is actually *in contradiction* with it, that's not a reductionist theory. If you want to give up reductionism, tell me where it breaks down and why.

    The first comment which you seem to refer to is this:

    "The only clash I can think of between some form of reductionism and quantum mechanics specifically - i.e. entangled systems not admitting maximally complete descriptions as independent entities despite their physical separation - is unequivocally problem-free, in the sense that this kind of reductionism has been empirically disproven for at least a few decades now. No big deal, if you ask me, because physics is still local."

    No, this has nothing to do with what I am talking about. As it says in the title of this blogpost, I am talking about the measurement problem.

    ReplyDelete
  141. - Sabine wrote -

    I already said this several times, here we go once again: A reductionist theory is one in which macroscopic concepts can be derived from the microscopic ones. If you have a postulate (measurement postulate) that does not only not derive from the microscopic law, but is actually *in contradiction* with it, that's not a reductionist theory. [...] If you want to give up reductionism, tell me where it breaks down and why

    --

    Sabine, please stop deflecting: repeating the *same* thing over and over does not equate to an explanation... and I already argued that, with the understanding I have of the words “reductionism” and “microscopic/macroscopic”, your first sentence applies to QM as much as it does to CM.

    Also *completely* unclear is what you mean with your second sentence.

    First, you can’t “derive” postulates from anything by definition

    Second, the postulate applies to all systems and all interactions (not just the macroscopic ones!), and is not in contradiction with anything I am aware of.

    Third, your claim that this breakdown of reductionism would be a problem, comes without a shred of a hint of what this problem would lead to: “show me where it breaks down” is undoable until you define “reductionism”more carefully and, besides, “reductionism” may well just be some other ism that can be ruled out as useful descriptor for Nature, as my elementary/composite example shows.

    Sabine also wrote -

    The first comment which you seem to refer to is this

    -

    No, I am referring to my first reply on Aug16th, whose main conclusion is that your entire blogpost should be retracted.

    Anyway, if I can venture into speculations about what is going on in your head, you seem to believe that the measurement postulate asserts the existence of something.

    Well, it does NOT: it is simply a rationalisation of how things are observed to interact. The only know rationalisation that accounts, for example, for the FACT that reducing continuously the strength of an interaction may not ultimately lead to a continuous reduction of the magnitude of the response, but to a continuous reduction of the *frequency* of one and the same *full magnitude* response (vs no response *at all*)

    While this behaviour is, I hope obviously, incompatible with any local classical model, there is absolutely *no difference* between the meaning of, say, “measuring a position coordinate” in QM and that of the same sentence in CM, despite what you seem to believe.

    What *is* different, of course, is that in QM you can’t even *define the meaning* of knowing *both* coordinates and momenta, but how can one think that this spoils the meaning of knowing something that CAN be measured is beyond me: the “quality” of this knowledge we are talking about (arrays of numbers) is identical in the two frameworks!

    If you have issues with “knowledge” is not because of QM. As a matter of fact, QM arose as soon we have been able to resolve finely enough the epistemologically necessary split of Nature into system + detector/observer (+ environment, if both the system and detector/observer are finite), that is the cornerstone of any scientific reasoning, because it is required by repeatability.

    What we found (the existence of some -many- sets of “finitely different” configurations, with no “interpolating” options *ever* observed) is inescapable. The very first paragraph of Dirac’s Principle (1930!) has all of this spelled out beautifully.

    Well, I made most of the argument again. I could keep going and proactively deconstruct another very common misconception about entangled state, but I'm probably approaching the length limit.

    Questions? Happy to expand

    ReplyDelete
  142. Sabine wrote:

    “If you define "detector" as something that does not make a measurement, then you have no problem with the measurement that such a detector makes. That's correct.”

    My detector does make a measurement. It does so without collapsing the wave-function.

    You seem to insist that a measurement requires a wave-function collapse. I think that we both agree that the wave-function collapse cannot be derived from the existing quantum mechanics formalism. Moreover, I don’t see how quantum mechanics could be extended in a consistent manner to include a wave-function collapse.

    But the wave-function collapse is part of the Copenhagen interpretation. In MWI there is no collapse and therefore there is no inconsistency.

    ReplyDelete
  143. Andrei wrote:

    “Now, if MWI is true, it should be possible to deduce both 1 and 2 because there is simply no other fundamental structure available, right?”

    This sounds like a nice argument but it does not work.

    Simulating a scientist in quantum mechanics seems like a hard problem, but we actually know the results of the calculation. The result is that all allowable outcomes of the simulated experiment are possible. To see which outcome is most probable we need to use the Born rule.

    For example, think of a scientist doing 10 spin up/down experiments. All 2^10=1024 outcomes are possible. The simulated scientist that saw 5 spins up would write that the chances of spin up are 50% and the one that saw 10 spins up would write 100%. The only way to figure out which outcome is most probable is to use the Born rule.

    ReplyDelete
  144. Andrea,

    I already answered these questions earlier and told you why what you say is wrong. I do not think this conversation is going anywhere and have no interest continuing it.

    ReplyDelete
  145. Udi,

    "My detector does make a measurement. It does so without collapsing the wave-function."

    Then it's not a detector.

    "You seem to insist that a measurement requires a wave-function collapse."

    I insist that a theory describes what we observe.

    ReplyDelete

COMMENTS ON THIS BLOG ARE PERMANENTLY CLOSED. You can join the discussion on Patreon.

Note: Only a member of this blog may post a comment.